UWorld Miscellaneous Final GMD
A 6-month-old boy is brought to the clinic for a wellness visit. The mother states he has been doing well since his last visit and has had no recent illnesses. The patient was born at term and has no chronic medical conditions. He takes no medications aside from vitamin D supplementation. The patient had been exclusively breastfed. The mother recently began introducing pureed fruits and vegetables. The patient continues to breastfeed but does not consume iron-fortified cereal. Vital signs are normal. The anterior fontanelle is open and flat. The oropharynx is clear. The neck is supple, and cardiopulmonary examination is unremarkable. The abdomen is soft with no organomegaly. It is explained to the mother that the patient is at risk for developing iron deficiency anemia. If no dietary changes are made, which of the following laboratory values is most likely to decrease first in this patient? Ferritin Mean corpuscular hemoglobin concentration Mean corpuscular volume Reticulocyte count Transferrin saturation
Beginning around age 6 months, term infants are at risk for developing iron deficiency. They are typically born with enough iron required for the first few months of life. As iron is lost steadily by shedding epithelium (eg, skin, intestinal tract), it must be replenished through dietary intake; newborns also require additional iron to support rapid body growth. Breastmilk contains only a low concentration of iron. Therefore, breastfed infants who do not receive additional iron-rich foods are at risk for developing iron deficiency. Iron deficiency progresses through the following stages: Decreased iron storage: When there is a net negative iron balance, iron is first removed from the storage pool (eg, hepatocytes, reticuloendothelial system); iron is stored intracellularly within ferritin complexes. Ferritin is also found in the blood, and serum levels generally fluctuate in proportion to iron stores. Therefore, the body's depletion of storage iron can be detected through a low serum ferritin level. Iron-limited erythropoiesis: As iron stores become depleted, serum iron levels decrease, followed by a compensatory rise in transferrin and a resultant decrease in transferrin saturation (Choice E). During this phase, there is iron-limited erythropoiesis, in which newly made erythrocytes (ie, reticulocytes) are iron deficient and reticulocyte production may begin to be limited. Because mature erythrocytes have a slow turnover (ie, 120 days), most cells sampled during this stage still have normal morphology, and the total hemoglobin value may still be normal. Iron deficiency anemia: If iron deficiency continues, the last phase is frank iron deficiency anemia, which is reflected by a low hemoglobin level with microcytosis (low mean corpuscular volume [MCV]) and hypochromia (low mean corpuscular hemoglobin concentration [MCHC]) in most erythrocytes (Choices B and C). Reticulocyte production decreases, resulting in an inappropriately low number of reticulocytes (Choice D). Therefore, a patient in the earliest stages of iron deficiency anemia would likely have only decreased ferritin with an otherwise normal transferrin saturation, MCV, MCHC, and reticulocyte count. Educational objective: Infants who do not receive adequate iron supplementation are at risk for developing iron deficiency, which progresses in stages as total body iron is depleted. The earliest stage involves loss of iron in storage (decreased serum ferritin), followed by iron-limited erythropoiesis with decreased serum iron and transferrin saturation. Frank iron deficiency anemia, involving the classic findings of low hemoglobin, microcytic/hypochromic erythrocytes, and a low reticulocyte count, is a late finding.
A 24-year-old man comes to the oncology clinic for a follow-up visit. The patient was diagnosed with Hodgkin lymphoma 4 months ago and received combination chemotherapy. He has had a nonproductive cough and shortness of breath with exertion over the past 2 weeks. On examination, there is no jugular venous distension or pedal edema. Heart sounds are normal with no murmurs. Lung auscultation reveals bilateral fine inspiratory crackles. Chest radiograph shows bilateral reticular interstitial opacities. Pulmonary function testing shows a restrictive pattern with decreased diffusion capacity for carbon monoxide. Chest x-ray and pulmonary function testing prior to the initiation of chemotherapy were normal. Which of the following is the most likely mechanism of action of the medication causing this patient's current symptoms? Disrupts microtubule polymerization causing M-phase arrest Impairs DNA synthesis by inhibiting thymidylate synthase Induces free radical formation and DNA strand breaks Inhibits topoisomerase II leading to DNA strand breaks Inhibits tyrosine kinases and cellular replication Produces cell-cycle arrests and apoptosis by inhibiting proteasomes
Bleomycin is commonly used in the treatment of Hodgkin lymphoma and germ cell tumors; it exerts an antineoplastic effect by binding with iron and oxygen molecules to create free radicals that cause DNA strand breaks. Lung toxicity, usually in the form of pulmonary fibrosis, is the most clinically important adverse effect of the drug. Toxicity from bleomycin likely results from oxidative free-radical damage to healthy tissue, leading to a cycle of tissue damage and repair. The lungs are particularly susceptible because they lack bleomycin hydrolase, an enzyme that helps inactivate bleomycin in other tissues. Patients who develop lung toxicity typically experience dyspnea, nonproductive cough, and crackles on lung auscultation within 6 months following initiation of therapy. Bibasilar interstitial opacities are typically seen on chest x-ray, and spirometry demonstrates reduced FVC with relatively preserved FEV1, consistent with a restrictive pattern. In addition, there is reduced diffusion capacity for carbon monoxide (DLCO) due to impaired gas exchange across an inflamed or fibrosed interstitium. (Choice A) Vinka alkaloids (eg, vincristine, vinblastine) bind to beta-tubulin to disrupt microtubule polymerization and prevent mitotic cell division. Peripheral neuropathy is a common adverse effect of these drugs. (Choice B) 5-fluoruracil inhibits thymidylate synthase to impair DNA synthesis. Bone marrow suppression (eg, anemia, neutropenia) can occur. (Choice D) Etoposide inhibits topoisomerase II to cause DNA strand breaks, and is commonly associated with bone marrow suppression. Anthracyclines (eg, doxorubicin) also exert at least part of their antineoplastic effect by binding to topoisomerase II and can cause cardiotoxicity that manifests as dilated cardiomyopathy. (Choice E) The tyrosine kinase inhibitors imatinib and erlotinib are used to treat chronic myelogenous leukemia and lung cancer, respectively. These drugs disrupt cellular replication and commonly cause diarrhea and skin rash. (Choice F) Proteasome inhibitors (eg, bortezomib) are used in the treatment of multiple myeloma. These drugs can cause bone marrow suppression and sometimes peripheral neuropathy. Educational objective: Bleomycin is an antineoplastic agent that binds to iron and oxygen molecules to create free radicals that cause DNA strand breaks. The most clinically important adverse effect of bleomycin is lung toxicity, usually manifesting as pulmonary fibrosis. Patients develop dyspnea and nonproductive cough, and pulmonary function testing shows a restrictive pattern with reduced diffusion capacity for carbon monoxide (DLCO).
A 2-day-old boy is being examined in the newborn nursery prior to discharge from the hospital. He was born at 38 weeks gestation by vaginal delivery. The pregnancy and delivery were uncomplicated, and the boy has been breastfeeding, stooling, and urinating normally. The patient's mother has beta-thalassemia trait, and his father has a normal hemoglobin electrophoresis. Vital signs and physical examination are normal. Which of the following hemoglobin compositions is most likely predominant in this infant? Alpha2Beta2 Alpha2Gamma2 Alpha2Beta2 Beta4 Gamma4
Hemoglobin is a tetramer that consists of 2 pairs of globin chains (total of 4 chains per molecule). During the first few weeks of embryogenesis, hemoglobin is synthesized by the yolk sac and contains zeta or epsilon globin chains (Choices G and F). Thereafter, one pair of the globin chains should always be alpha and the other should be non-alpha. Fetal hemoglobin (Hb F) production begins around 8 weeks gestation and replaces all embryonic hemoglobin by 14 weeks gestation, when erythropoiesis in the fetal liver and spleen is established. Hb F consists of 2 alpha and 2 gamma protein subunits (α2γ2). Production declines at birth, and Hb F comprises ~60-80% of all hemoglobin in a term newborn. Hb F is gradually replaced by adult hemoglobin (Hb A, α2β2) (Choice A) during the first 6 months of life, after which Hb A composes the vast majority of adult hemoglobin. Compared to red blood cells with Hb A, those with Hb F have a high oxygen affinity as Hb F binds to 2,3-bisphosphoglycerate poorly. The greater affinity of Hb F facilitates transplacental oxygen delivery from the maternal circulation to that of the fetus. (Choice C) Hemoglobin A2 (α2δ2) is a normal hemoglobin variant that makes up 2%-3% of hemoglobin in a healthy adult and is functionally similar to Hb A. Patients with beta-thalassemia major have impaired beta globin production, resulting in an excess of alpha globin chains (eg, Hb A2, Hb F) and no Hb A. (Choices D and E) Alpha-thalassemia results from a shortage of alpha globin chains. Hemoglobin H (β4) and hemoglobin Barts (γ4) have a very high oxygen affinity and cannot release oxygen, resulting in tissue hypoxia. Hemoglobin H disease manifests as chronic hemolytic anemia. Hemoglobin Barts is incompatible with life (eg, hydrops fetalis) as normal fetal and adult hemoglobin cannot be produced. Educational objective: Hemoglobin F (Hb F) is the predominant hemoglobin type in the second and third trimesters of pregnancy and during the first few months after birth. Hb F consists of 2 alpha and 2 gamma protein subunits (α2γ2) and has a high affinity for oxygen, which facilitates oxygen transport across the placenta to the fetus. Hb A (α2β2) is the major hemoglobin in adults.
A 68-year-old man is brought to the emergency department due to sudden-onset, severe right lower extremity pain. On physical examination, the right leg is pale and cold to the touch with absent distal pulses. Cardiac auscultation reveals an irregular tachycardia, and ECG shows atrial fibrillation. Right lower extremity ultrasonography reveals an arterial embolus. Intravenous heparin infusion is initiated. This medication is most likely to cause which of the following coagulation changes? TT, PTT, Xa activity no change, +, - no change, +, + +, no change, + +, +, no change +, +, -
Heparin is an endogenous anticoagulant that exerts its effect indirectly by binding to antithrombin III (AT), a serine protease inhibitor that irreversibly neutralizes clotting factors. Heparin binding alters the shape of AT, which converts it from a slow to rapid inactivator of clotting factors and increases its activity >1,000 fold. Although AT neutralizes a number of clotting factors in the intrinsic pathway, its anticoagulant effect primarily stems from the inactivation of factor Xa and thrombin. Therefore, patients who receive heparin products typically have the following coagulation changes: Prolonged thrombin time: Thrombin inactivation is strongest with unfractionated heparin infusion because the length of heparin chains in this formulation are longer, which inactivates thrombin more rapidly. Although low-molecular-weight heparin formulations contain shorter chain heparin molecules, they also inactivate thrombin but to a lesser extent. Decreased activity of factor Xa: All forms of heparin (eg, unfractionated, low molecular weight, fondaparinux) have strong activity against factor Xa. Antifactor Xa activity is usually monitored to determine when adequate anticoagulation is achieved. Prolonged partial thromboplastin time (PTT): Heparin primarily inhibits the intrinsic and final common clotting pathway; therefore, PTT is prolonged. Unlike heparin medications, which exert their anticoagulation effects broadly through the activation of AT, direct oral anticoagulants inhibit a single enzyme in the coagulation cascade, such as thrombin (eg, dabigatran) or factor Xa (eg, rivaroxaban). Educational objective: Heparin is an indirect anticoagulant that alters the shape of antithrombin (AT) III, thereby converting it from a slow to rapid inactivator of clotting factors. Because AT primarily neutralizes factor Xa and thrombin, patients on heparin have decreased factor Xa activity and prolonged thrombin time/partial thromboplastin time. In contrast, direct oral anticoagulants specifically target a single enzyme in the coagulation cascade, such as thrombin (eg, dabigatran) or factor Xa (eg, rivaroxaban).
A 41-year-old woman with a long history of heavy menstrual bleeding comes to the office for follow-up. Two weeks ago, the patient was diagnosed with iron deficiency anemia and was started on oral ferrous sulfate. Today, vital signs and physical examination are unremarkable. Laboratory evaluation shows an appropriate response to the medication with an increase in reticulocyte production. She is instructed to continue iron supplementation. The woman is concerned about iron buildup and damage to vital organs if she continues to take the medication. She is reassured that iron homeostasis is closely regulated; as the body's iron stores increase, a hormone is released and functions in reducing the amount of dietary iron absorbed from the intestines. Which of the following cells is most likely responsible for secreting this hormone? Bone marrow macrophages Bone marrow stem cells Hepatic parenchymal cells Intestinal epithelial cells Renal tubular cells
Hepcidin is an acute phase reactant synthesized by hepatic parenchymal cells that acts as the central regulator of iron homeostasis. High iron levels and inflammatory conditions increase the synthesis of hepcidin, while hypoxia and increased erythropoiesis act to lower hepcidin levels. Hepcidin influences body iron storage through its interaction with ferroportin, a transmembrane protein responsible for transferring intracellular iron to the circulation. Upon binding hepcidin, ferroportin is internalized and degraded, decreasing intestinal iron absorption and inhibiting the release of iron by macrophages. Regulation of intestinal iron absorption is crucial for maintaining iron homeostasis, since blood loss is the only way of removing large amounts of iron from the body. Iron absorption from the proximal small intestine is facilitated by the divalent metal transporter-1 (DMT-1). Once inside the intestinal cells, iron may take 1 of 2 paths: Iron may bind to ferritin (a primarily intracellular iron-binding protein) and remain stored within the enterocyte. The stored iron is excreted in the stool as enterocytes slough off and are replaced Iron may enter the circulation through ferroportin, the basolateral iron transporter of the enterocyte. Free iron released into the circulation is transported throughout the body by transferrin (an iron-binding transport protein), which becomes internalized after interacting with transferrin receptors present on all cells (Choices A and B) When senescent red blood cells are destroyed by macrophages, the iron is recycled by the reticuloendothelial system for use in new erythroid precursors. Iron released by macrophages under the governance of hepcidin binds rapidly to transferrin and is delivered to the bone marrow, where erythroid precursors use it to synthesize hemoglobin. This internal iron cycle conserves the body's iron very efficiently, and only small amounts of additional iron are required for daily red cell production in the absence of significant blood loss. (Choice D) Enterocytes store iron for excretion in the feces or transport it into the circulation depending on circulating levels of the polypeptide hormone hepcidin. Lower hepcidin levels favor iron absorption and accumulation in the body. (Choice E) Renal tubular cells play a limited role in iron metabolism by secreting lactoferrin, which binds to free iron in the urine and possibly recovers it for metabolic use. Educational objective: Hepcidin is synthesized by the liver that acts as the central regulator of iron homeostasis. High iron levels and inflammatory conditions increase the synthesis of hepcidin; hypoxia and increased erythropoiesis act to lower hepcidin levels. Low hepcidin levels increase intestinal iron absorption and stimulate iron release by macrophages.
In vitro studies demonstrate that tumor cell lines can become resistant after exposure to various anticancer agents. These cells express a specific cell surface glycoprotein that has which of the following functions? Tyrosine kinase-coupled receptor Adenylate cyclase-coupled receptor Cell adhesion molecule Transmembrane ion channel ATP-dependent transporter
Human tumor cells have developed the ability to resist chemotherapy in much the same way that many bacteria have developed resistance to antibiotics. One well-studied mechanism by which human tumor cells resist these agents is via the human multidrug resistance (MDR1) gene. The prototype product of this gene is P-glycoprotein, a transmembrane protein that functions as an ATP-dependent efflux pump. P-glycoprotein is normally expressed in intestinal and renal tubular epithelial cells and functions to eliminate foreign compounds from the body. This protein is also present in the capillary endothelium of the vessels that form the blood-brain barrier. In this location, P-glycoprotein prevents the penetration of foreign compounds into the CNS. In tumor cells, this ATP-powered transmembrane pump protein actively removes chemotherapeutic agents, particularly hydrophobic agents like the anthracyclines. Drugs such as verapamil, diltiazem, and ketoconazole, among others, have been shown to reduce the action of this multidrug resistance protein in experimental models; the development of agents specifically intended to abolish this mechanism of resistance is currently underway. (Choice A) Tyrosine kinase receptors generally mediate the effects of hormones that promote anabolism and cell growth such as insulin, insulin-like growth factor 1, epidermal growth factor, platelet-derived growth factor, and others. (Choice B) cAMP is the second messenger in the G-protein / adenylate cyclase second messenger system. This system is utilized by numerous hormone receptors including beta-adrenergic receptors, V2 ADH receptors, and calcitonin, glucagon, TSH, ACTH, and HCG receptors, among many others. (Choice C) Cell adhesion molecules (CAMs) are proteins located on the cell surface that mediate binding with other cells or with the extracellular matrix. Examples include selectins, integrins, and cadherins. Cell adhesion molecules are generally downregulated in malignant tumors, which allows these cells to spread from their site of origin. (Choice D) Transmembrane ion channels selectively allow certain ions to pass into or out of the cell. These channels facilitate the formation of a voltage gradient across the cell membrane. Examples of ion channels, transporters, and exchangers are ubiquitous. These proteins participate in the formation of urine, nerve and myocyte depolarization, muscle contraction, sweat formation, exocrine secretion formation, and many other processes. Educational Objective: The human multidrug resistance (MDR1) gene codes for P-glycoprotein, a transmembrane ATP-dependent efflux pump protein that has a broad specificity for hydrophobic compounds. This protein can both reduce the influx of drugs into the cytosol and can increase efflux from the cytosol, thereby preventing the action of chemotherapeutic agents.
An infant born to a Greek immigrant appears healthy at birth but develops transfusion-dependent hemolytic anemia by the age of 6 months. His erythrocytes contain insoluble aggregates of hemoglobin subunits. The child developed normally in utero because at that time he produced high quantities of: Alpha-globin Beta-globin Gamma-globin Delta-globin
In adults, hemoglobin A is the predominant form of hemoglobin. It is a tetramer consisting of two alpha and two beta chains. Normally the synthesis of alpha and beta chains is tightly regulated such that one α-chain is synthesized for every β-chain. Hemoglobin formation begins within a few weeks of conception. The initial hemoglobin formed by a fetus in utero is called embryonic hemoglobin (Gower). This hemoglobin is composed of two zeta (ζ) and two epsilon (ε) chains (ζ2ε2) and is produced in the embryonic yolk sac. Within a few weeks the fetal liver starts synthesizing hemoglobin F (fetal hemoglobin). This form of hemoglobin is composed of two alpha and two gamma chains (α2γ2). HbF is the major hemoglobin in the fetus during last few months of gestation and in infants during first few weeks of postnatal life. HbA synthesis starts during the final month of gestation and gradually replaces HbF during postnatal life. Knowing the chronology of fetal hemoglobin formation and the gradual transition to adult hemoglobin (HbA) is important in understanding the relationship between clinical manifestations and postnatal age in beta thalassemia. Thalassemias are hereditary hemolytic anemias resulting from defective synthesis of globin chains. As described above, the synthesis of alpha and beta globin chains is very coordinated. In patients with thalassemia, the synthesis of either alpha or beta chains is defective. Beta thalassemia is caused by defective synthesis of beta chains. There are two copies of the beta globin gene (one from each parent). If only one gene is defective the patient will have beta thalassemia trait (beta thalassemia minor) and lack significant anemia. A defect in both beta globin genes results in severe hemolytic anemia known as beta thalassemia major. In this disease, alpha chains are produced normally but they cannot form stable tetramers due to the lack of beta globin chains. This failure to form stable hemoglobin leads to precipitation of alpha globin chains and premature lysis of red blood cells. Beta thalassemia cannot become symptomatic as long as there are significant amounts of gamma chains present because gamma chains make up for the absence of HbA beta chains in forming tetramers. Thus, in late gestation and early postnatal life, the expression of hemoglobin A is offset by gamma chain production (Choice C). As gamma chain production wanes, patients will become symptomatic. (Choice A) α-globin is a normal component of both HbA and HbF. α-globin synthesis is defective in α-thalassemia but it is normal in β-thalassemia. α-globin is able to combine with γ-globin to form HbF. This allows patients with β-thalassemia major to be asymptomatic in utero and in the first few months following birth. (Choice B) β-globin synthesis is defective in patients with b-thalassemia. (Choice D) δ-globin is a minor globin gene that is expressed at very low levels in normal adults. Two α-globins and two δ-globins combine to form hemoglobin A2. (Choice E) ζ-globin is a component of hemoglobin Gower, the initial hemoglobin formed by the embryo very early in embryogenesis. Educational Objective: HbF contains γ-globin instead of β-globin. Patients with homozygotic β-thalassemia (β-thalassemia major) are asymptomatic at birth due to the presence of γ-globins and HbF. Switching to HbA production and the cessation of γ-globin synthesis precipitates the symptoms of β-thalassemia.
A 5-year-old boy is brought to the office by his parents to establish medical care. The family recently immigrated to the United States from Nigeria. Both parents have sickle cell trait, and the patient was diagnosed with homozygous sickle cell disease (HbSS) at the age of 6 months. He has had several prior vasoocclusive crises and takes folic acid and hydroxyurea. Physical examination shows no abnormalities. Hemoglobin electrophoresis in this patient is most likely to reveal which of the following predominant hemoglobin patterns? HbS and HbF HbS, HbA, and HbA2 HbS, HbA, HbA2, and HbF HbS, HbA, and HbF HbS, HbC, and HbF
In healthy children and adults, the predominant hemoglobin is hemoglobin A (HbA), which contains a pair of alpha and beta globulins. However, small quantities of other hemoglobin types are also present; these include hemoglobin A2 (HbA2), which contains a pair of alpha and delta globulins, and hemoglobin F (HbF), which contains a pair of alpha and gamma globulins. In homozygous sickle cell disease (sickle cell anemia), there are point mutations in both beta globulin genes that substitute valine for glutamate at position 6. This generates a new type of hemoglobin called hemoglobin S (HbS), which is composed of a pair of alpha and a pair of beta sickle globulins. HbS polymerizes when deoxygenated, reducing erythrocyte deformability and promoting painful vasoocclusion and chronic hemolysis. Because patients with sickle cell anemia have mutations in both beta globulin genes, they are unable to produce HbA (Choices B, C, and D). However, forms of normal hemoglobin that do not contain beta globulin (eg, HbF, HbA2) are present at normal or mildly increased levels. Raising the concentration of these normal hemoglobins is particularly helpful because it dilutes HbS, preventing it from polymerizing. Therefore, hydroxyurea, which increases HbF production, is often administered to limit symptoms and complications of the disease. (Choice E) Patients with hemoglobin SC disease have 1 beta globulin with the sickle mutation and 1 beta globulin with a mutation that substitutes lysine for glutamate at position 6. They produce two predominant hemoglobins: HbS and Hemoglobin C, the latter of which is not seen in sickle cell anemia as there is no hemoglobin C mutation. Educational objective: Homozygous sickle cell disease is marked by a point mutation in both beta globulin genes. This leads to the generation of a new predominant hemoglobin called hemoglobin S (HbS), which polymerizes when deoxygenated. Because hemoglobin F (HbF) prevents HbS polymerization, hydroxyurea is often administered to increase HbF levels and reduce complications of the disease.
A 23-year-old woman is evaluated due to 10 days of nonproductive cough, low-grade fever, headache, and malaise. The patient has no other medical problems and takes no medications. Lung examination reveals scattered rales. Chest x-ray reveals bilateral patchy areas of consolidation. She has mild anemia and an elevated serum lactate dehydrogenase level. The patient is treated for presumed Mycoplasma pneumonia with azithromycin. Two months later, her symptoms and the anemia have resolved. Which of the following best explains the resolution of this patient's anemia? Elimination of bacterial cell wall antigens Elimination of intraerythrocytic microorganisms Fading of immune response against bacteria Replenishment of body iron stores Replenishment of intracellular enzyme stores
Mycoplasma pneumoniae is a common cause of tracheobronchitis and walking pneumonia. It attacks the respiratory epithelium by binding to an oligosaccharide (I-antigen) that is also present on the surface of erythrocytes. This leads to the formation of cross-reacting IgM antibodies that attach to red blood cells, activate the complement system, and cause erythrocyte lysis. The cross-reacting antibodies are called cold agglutinins because they bind to erythrocytes in areas where the blood temperature is below core body temperature (eg, distal extremities, nose). Most patients with cold agglutinins are asymptomatic, but some develop manifestations of intravascular hemolytic anemia such as elevated reticulocyte count and lactate dehydrogenase level. IgM titers usually begin to fall approximately 4 weeks after initial infection leading to a resolution of the hemolytic anemia (within 8 weeks). Other extrapulmonary manifestations of M pneumoniae include Stevens-Johnson syndrome, joint pain, encephalitis, cardiac rhythm disturbances, and bullous myringitis. (Choice A) M pneumoniae is an atypical bacterium; it has a plasma membrane composed of cholesterol (unlike most bacteria) and no cell wall. The lack of a cell wall makes this pathogen resistant to beta-lactam antibiotics. (Choice B) Malaria and babesiosis are associated with intraerythrocytic organisms that can cause cell lysis and anemia. However, these pathogens usually cause high fever and more severe clinical syndromes. In addition, azithromycin monotherapy is not an effective cure for these infections. (Choice D) Iron deficiency anemia, which is most commonly due to blood loss, can often be improved by replenishing body iron stores (eg, oral iron supplementation). Chronic infection or inflammatory conditions can be associated with anemia of chronic disease, which is due to impaired utilization of iron (not absolute iron deficiency). (Choice E) Enzyme deficiencies that cause anemia include glucose-6-phosphate dehydrogenase (G6PD) deficiency and pyruvate kinase deficiency. Infection and medications that cause oxidative stress (eg, sulfonamides, antimalarial agents) can cause hemolysis in patients with G6PD; however, G6PD is an X-linked disorder (it is seen in men). Educational objective: Mycoplasma pneumoniae binds an oligosaccharide on the respiratory epithelium that is also present on erythrocytes, leading to the generation of cross-reacting IgM antibodies (cold agglutinins). Patients with M pneumoniae infections often develop mild, transient hemolytic anemia that resolves as IgM antibody titers decline (6-8 weeks after infection begins).
A 66-year-old man with hypertension and stage IV chronic kidney disease comes to the office for follow-up. The patient has had worsened fatigue and low energy over the past several months but no chest pain or shortness of breath. He was previously nonadherent with antihypertensive therapy but has been following treatment recommendations since renal dysfunction was diagnosed 2 years ago. Blood pressure is 126/78 mm Hg and pulse is 68/min. Cardiac auscultation reveals an S4 sound. The rest of the physical examination is unremarkable. Laboratory results show normocytic, normochromic anemia with a low reticulocyte count. Serum iron studies are within normal limits. Treatment with a recombinant glycoprotein hormone is begun, and repeat laboratory testing several weeks later shows an improvement in hemoglobin level. The effects of the hormone prescribed for this patient are most likely mediated by which of the following pathways? Adenylate cyclase/cyclic AMP Arachidonic acid/phospholipase Janus kinase 2/signal transducers and activators of transcription Nuclear receptors Phosphatidylinositol 3-kinase Ras/mitogen-activated protein kinase
Patients with chronic kidney disease (CKD) often develop normocytic anemia as the glomerular filtration rate declines, due to an inability to generate adequate quantities of erythropoietin (EPO). EPO is a glycoprotein produced by peritubular fibroblast cells in the renal cortex in response to tissue hypoxia (as seen with anemia). Healthy individuals increase EPO production up to 10,000-fold in response to anemia, but patients with CKD have chronic renal inflammation that damages EPO-producing cells, thereby preventing an adequate EPO response. Therefore, patients with CKD are often treated with recombinant EPO agents (eg, epoetin, darbepoetin) to supplement EPO production. EPO travels from the kidney through the bloodstream to the bone marrow, where it binds to a surface receptor on erythrocyte colony-forming unit cells (erythrocyte precursors). Binding activates the Janus kinase 2/signal transducers and activators of transcription (JAK2/STAT) signaling pathway, which induces the transcription of proteins that inhibit erythrocyte precursor apoptosis and promote survival. Binding also activates signals that increase the differentiation of erythrocyte precursors into mature erythrocytes. Together, these effects increase the production of erythrocytes and improve circulating red blood cell counts. Activating mutations affecting the JAK2 gene are associated with polycythemia vera, a chronic myeloproliferative disorder marked by the clonal proliferation of erythrocytes (leading to dramatic elevations in hematocrit). (Choice A) Cyclic AMP mediates the effects of a variety of hormones including glucagon, antidiuretic hormone (V2 receptor), and corticotropin. However, this pathway is not a primary intermediate in erythrogenesis. (Choice B) Cleavage of membrane phospholipids by phospholipase A2 leads to the formation of arachidonic acid, which is in turn converted into prostaglandins and leukotrienes that help regulate the inflammatory response. A variety of medications directly or indirectly affect this system, including glucocorticoids, nonsteroidal anti-inflammatory drugs, and leukotriene inhibitors. (Choice D) Widely used therapeutic hormones that activate a nuclear receptor include thyroid hormone, vitamins A & D, and the steroid hormones (eg, glucocorticoids, mineralocorticoids, sex hormones). (Choice E) Therapeutic hormones that activate a phosphatidylinositol second messenger system include antidiuretic hormone (V1 receptor), gonadotropin-releasing hormone, and oxytocin. (Choice F) The Ras/mitogen-activated protein kinase signal transduction system is used by a variety of growth factors (eg, epidermal growth factor, platelet-derived growth factor), which primarily act to stimulate cell proliferation. Uncontrolled activation of this system can lead to malignancy. Educational objective: Chronic kidney disease is commonly associated with normocytic anemia due to reduced production of erythropoietin by the kidneys. Erythropoietin signal transduction is primarily mediated by the Janus kinase 2/signal transducers and activators of transcription (JAK2/STAT) pathway, which promotes erythrocyte precursor survival.
A 52-year-old man comes to the physician complaining of dizziness, headaches, and pruritus after showering. He has smoked half a pack of cigarettes daily for the past fifteen years and drinks alcohol socially. Physical examination shows a reddish facial complexion and mild splenomegaly. Laboratory results are as follows: Complete blood count Hemoglobin 19 g/dL Hematocrit 59% Erythrocytes 7.5 million/µL Platelets 550,000 /µL Leukocytes 15,600 /µL Which of the following is the most likely cause of this patient's findings? Decreased circulating plasma volume Decreased blood oxygen saturation Increased bone marrow sensitivity to growth factors Increased red blood cell life span Increased serum erythropoietin levels
Polycythemia vera (PV) is a clonal myeloproliferative disease of pluripotent hematopoietic stem cells. Approximately 95% of patients with PV have a V617F mutation involving the JAK2 gene, which is involved in signal transduction. This mutation replaces a valine with phenylalanine at the 617 position, rendering hematopoietic cells more sensitive to growth factors such as erythropoietin and thrombopoietin. PV presents with increased RBC mass, increased plasma volume, and low erythropoietin levels. Additional manifestations can include an elevated platelet and/or WBC count, thrombotic events (from blood hyperviscosity), peptic ulceration and aquagenic pruritus (due to histamine release from basophils), and gouty arthritis (from increased cell turnover). Physical examination typically shows a plethoric, reddened face and splenomegaly. Diagnosis is established by confirming low serum erythropoietin levels and bone marrow aspiration cytogenetic studies showing a JAK2 mutation. Treatment involves serial phlebotomy as necessary to keep the hematocrit < 45%. (Choice A) Dehydration or excessive diuresis can also cause elevated hematocrit, mild leukocytosis, and thrombocytosis due to low effective circulating volume. However, splenomegaly and symptoms such as pruritus would not be seen. (Choice B) Hypoxia is a strong stimulus for erythropoietin production. SaO2 < 92% (PaO2 < 65 mm Hg) appears to be the threshold for the development of (physiologic) secondary polycythemia. Conditions such as chronic obstructive pulmonary disease and obstructive sleep apnea can cause secondary polycythemia. However, these will not cause leukocytosis, thrombocytosis, or splenomegaly. (Choice D) Increased red cell life span would not be expected to cause leukocytosis and thrombocytosis. (Choice E) EPO-producing tumors (eg, renal cell carcinoma, hepatocellular carcinoma) can cause secondary polycythemia due to abnormal erythropoietin production. Workup will show an elevated erythropoietin level. However, the combination of multiple elevated cell lines and splenomegaly is unlikely to result from a process causing secondary polycythemia. Educational objective: Polycythemia vera (PV) is a clonal myeloproliferative disease characterize by an increased RBC mass, increased plasma volume, and low erythropoietin levels. PV can be differentiated from secondary polycythemia by the presence of leukocytosis, thrombocytosis, and/or splenomegaly. The majority of patients with PV have a JAK2 mutation rendering hematopoietic stem cells more sensitive to growth factors.
A 65-year-old man comes to the office due to several months of severe fatigue that limits his ability to do daily activities. He has also had upper abdominal fullness, early satiety, and unintentional weight loss but no fever, night sweats, cough, or rash. The patient has not seen a physician for many years. He does not use tobacco, alcohol, or illicit drugs and is a retired chemical engineer. Physical examination shows mucosal pallor with no scleral icterus. The lungs are clear on auscultation, and heart sounds are normal. There is mild hepatomegaly, and the spleen tip is palpable close to the midline of the abdomen. Laboratory evaluation reveals pancytopenia. Peripheral blood smear shows immature granulocytes, nucleated erythrocytes, and teardrop cells. Bone marrow aspiration is attempted but yields no marrow. Which of the following histopathological features is most likely to be present in this patient's spleen? Accumulation of macrophages with fibrillary cytoplasm Diffuse neutrophilic infiltration and follicular necrosis Dilated sinusoids and fibrous nodules with hemosiderin Large islands of hematopoietic progenitor cells Numerous noncaseating epithelioid granulomas
Primary myelofibrosis is a chronic myeloproliferative disorder marked by the clonal expansion of atypical megakaryocytes. These atypical cells produce transforming growth factor-beta, which stimulates fibroblasts in the bone marrow to lay down collagen, leading to subsequent bone marrow fibrosis. Because fibrotic bone marrow is inhospitable to hematopoietic stem and progenitor cells, these precursor cells migrate out of the bone marrow to the liver and spleen, the primary sites of extramedullary hematopoiesis. The proliferation of erythroid, myeloid, and megakaryocytic progenitor cells in the spleen leads to a marked expansion of the splenic red pulp and usually results in dramatic splenomegaly. Nucleated red cells and immature granulocytes spill into the circulation and can be identified on peripheral blood smear. Dacrocytes are also common due to erythrocyte membrane damage during red blood cell production in the spleen or fibrotic marrow. Because extramedullary hematopoiesis is less efficient than medullary hematopoiesis, patients typically have one or more cytopenias. The diagnosis of primary myelofibrosis requires bone marrow evaluation. Aspiration of bone marrow is frequently unsuccessful (dry tap) due to fibrosis. Therefore, bone marrow biopsy is generally required and usually shows hypocellular marrow with significant fibrosis and atypical megakaryocytes. A minority of primary myelofibrosis cases have been linked to chemical exposure (eg, toluene, benzene). (Choice A) Gaucher disease is an autosomal recessive lysosomal storage disease associated with hepatosplenomegaly and pancytopenia due to the accumulation of glucocerebroside within the cytoplasm of macrophages (crumpled-paper appearance). However, most cases present in childhood. In addition, a dry tap on bone marrow aspirate and dacrocytes on peripheral smear would be atypical. (Choice B) Acute infections of the spleen (eg, septic emboli from infective endocarditis) are typically associated with diffuse neutrophilic infiltration. Follicular necrosis is also common if the infection is due to group A Streptococcus. A dry tap, dacrocytes, and lack of fever make this less likely. (Choice C) Congestive splenomegaly due to portal hypertension (eg, from liver cirrhosis) is associated with dilated sinusoids, fibrosis of the red pulp, and hemosiderin-laden macrophages. A dry tap and dacrocytes would not be seen. (Choice E) Splenic noncaseating granulomas are common in sarcoidosis, some malignancies, and some infectious disorders. However, a dry tap and dacrocytes make primary myelofibrosis far more likely. Educational objective: Primary myelofibrosis is a chronic myeloproliferative disorder associated with bone marrow fibrosis. Extramedullary hematopoiesis leads to marked expansion of the splenic red pulp with hematopoietic progenitor cells, resulting in massive splenomegaly. Patients also usually have hepatomegaly, cytopenias, and peripheral smear abnormalities (eg, dacrocytes, nucleated red cells, immature granulocytes).
A 24-year-old African American woman comes to the office with her husband for prenatal counseling. She has a 3-year-old child with sickle cell anemia from a previous marriage, and the child's father died in a car accident. The patient remarried last year and is interested in having more children. She and her new husband do not have sickle cell anemia, and the patient's husband has no other children. However, the patient and her husband are worried that their future children could have sickle cell anemia. A urine pregnancy test is negative. Which of the following is the best initial test that can be offered to this couple? Chorionic villous sampling during future pregnancy Maternal hemoglobin electrophroesis Northern blot analysis of paternal blood sample Paternal hemoglobin electrophoresis Paternal karyotype analysis
Sickle cell anemia is an autosomal recessive hemoglobinopathy. Affected patients must inherit 2 mutant genes for hemoglobin S (HbS), one from each parent. In order for an offspring of 2 parents without sickle cell anemia to be affected, both parents must carry the sickle cell trait. Any offspring resulting from such pairings will have a 1 in 4 chance of having sickle cell disease (SCD). This patient is a carrier of the sickle cell trait as she has a child with sickle cell anemia. Hemoglobin electrophoresis, a type of gel electrophoresis, is used to determine if the patient's new husband also has sickle cell trait. Abnormal hemoglobin (eg, HbS), moves at a slower speed than normal hemoglobin due to the replacement of glutamic acid by valine. The husband's test results combined with the maternal family history will determine the risk that future offspring will inherit sickle cell anemia. (Choice A) The chorionic villi are part of the placenta and, as such, are fetal tissue. Once fetal DNA is obtained, DNA sequencing can be performed to evaluate for sickle cell mutation. Chorionic villus sampling (CVS) can be performed at 10-14 weeks gestation. However, it is relatively invasive and carries a risk of miscarriage, spotting/bleeding, infection, and other complications. CVS is not helpful to a patient who wishes to determine her risk prior to conception. (Choice B) It is not necessary to perform maternal hemoglobin electrophoresis as it has been established from this patient's history that she has sickle cell trait. (Choice C) Northern blots use electrophoresis to separate RNA by size, followed by the application of a DNA probe to detect a specific RNA molecule. This allows for the quantification of the RNA expression of specific genes. Hemoglobin electrophoresis is a more appropriate test for SCD as it involves the separation of protein (eg, hemoglobin) rather than RNA. (Choice E) A karyotype is a visual analysis of all the chromosomes in a cell ordered by size and stained to reveal chromosomal banding. SCD results from a single nucleotide mutation, not from any abnormality in overall chromosome structure. Disorders due to chromosomal abnormalities include Klinefelter syndrome (47,XXY), Turner syndrome (45,XO) and trisomy disorders. Educational objective: Sickle cell anemia is an autosomal recessive hemoglobinopathy. In order for a child to have sickle cell disease, both parents must be carriers. Hemoglobin electrophoresis can be used to determine the carrier status of a prospective parent who has no history of sickle cell anemia.
A 1-year-old African American boy is brought to the emergency department by his parents with a 3-hour history of severe swelling and tenderness of the hands and feet. They do not know what caused the swelling and say that the boy cries when his hands or feet are touched. The patient has no history of recent illness and is up to date on all immunizations. The boy attends day care, but his parents are unsure about any sick contacts. His family history is significant for an older brother who died of pneumococcal sepsis at age 6. Physical examination shows bilateral, severe swelling of the hands and feet. Which of the following is most likely abnormal in this patient? Left ventricular systolic function Serum albumin Serum C4 complement fraction Serum haptoglobin Systemic venous pressure Urine cortisol excretion
Sickle cell disease (SCD) is the most common autosomal recessive disorder in African Americans. Its manifestations include the following: Hemolysis: Repeated sickling of red blood cells leads to permanent deformation and premature erythrocyte destruction by macrophages and mechanical stress. This causes intra- and extravascular hemolysis, resulting in increased indirect bilirubin and lactate dehydrogenase and decreased haptoglobin. Haptoglobin binds circulating hemoglobin and reduces renal excretion of free hemoglobin, preventing tubular injury. Vasoocclusive symptoms: Patients with SCD may experience pain from hypoxic tissue injury and infarctions due to obstruction of small vessels by sickled cells. Microvascular occlusion typically involves the bone marrow, periosteum, and deep muscles but can occur in other organs. This patient is presenting with dactylitis (hand-foot syndrome), which results from small infarctions in the bones of the extremities. These infarctions cause swelling, tenderness, and warmth. Dactylitis is common in patients with SCD during the first few years of life as the affected bones still contain hematopoietic bone marrow. In older children and adults, vascular occlusion leads to pain crises, acute chest syndrome, leg ulceration, priapism, autosplenectomy, and stroke. Infections: Patients with SCD are predisposed to infections with encapsulated organisms (eg, Streptococcus pneumoniae) because repeated splenic infarcts cause functional asplenia. (Choices A and E) The chronic anemia in SCD can result in cardiomegaly due to chronically increased cardiac output (systolic function is largely preserved due to an increase in stroke volume). However, this typically does not occur until after infancy. Older patients can develop congestive heart failure due to ischemic cardiomyopathy and present with decreased cardiac output and increased systemic venous pressure. (Choices B and F) Serum albumin and urine cortisol excretion are typically normal in patients with SCD. (Choice C) Hereditary angioedema is a rare autosomal dominant disorder that is associated with painless episodes of swelling involving the face, lips, larynx, and extremities. It is caused by C1-inhibitor deficiency, not vasoocclusion. Educational objective: Dactylitis (painful swelling of the hands and feet) is a common presentation of sickle cell disease (SCD) in young children. It is one of many vasoocclusive manifestations of SCD. Sickling episodes result in hemolysis, which leads to increased indirect bilirubin and lactate dehydrogenase and decreased levels of haptoglobin.
A 26-year-old woman comes to the office for a follow-up appointment. She has a history of immune thrombocytopenia that failed to improve with glucocorticoid therapy. The patient underwent splenectomy 6 months ago, which resulted in improved platelet counts. She received the recommended vaccinations after her surgery and currently feels well. Physical examination shows no abnormalities. A peripheral blood smear reveals many red blood cells of a specific morphology as indicated by the arrow below. Target cells present An increase in which of the following features best explains the presence of these abnormal cells? Aggregated iron deposits Clusters of remnant DNA Denatured hemoglobin Fragility to osmotic stress Precipitated ribosomes Surface area-to-volume ratio
Target cells are red blood cells with a dark center, a surrounding halo of pallor, and a dark peripheral ring. They form in erythrocytes with an excessive surface area-to-volume ratio, which results in redundant erythrocyte cell membrane folding upon itself (thereby creating the appearance of a target). Target cells occur in the following conditions: Reduced erythrocyte cell volume from deficient hemoglobin synthesis (eg, thalassemia, iron deficiency) or structural mutations to hemoglobin (eg, sickle cell) Excessive erythrocyte cellular membrane due to greater cholesterol-to-phospholipid ratios (eg, obstructive liver disease) or splenectomy Macrophages in the red pulp of the spleen remove excess membrane from red cells, a process called "splenic conditioning." Patients who undergo splenectomy are unable to prune erythrocytes and generally develop target cells a few weeks following the procedure. However, over time, macrophages in nonsplenic tissue (eg, liver) take up the task of pruning erythrocytes, which typically eliminates (or diminishes) the presence of target cells in the circulation. (Choice A) Pappenheimer bodies are aggregated iron deposits in erythrocytes that are often seen in patients with sideroblastic anemia. They appear as dark blue inclusions. (Choice B) Howell-Jolly bodies are clusters of DNA remnants in erythrocytes due to hyposplenism or asplenism (the spleen usually removes the DNA remnants). They appear as peripheral, round, dark purple, blue, or red inclusions. (Choice C) Heinz bodies are aggregates of denatured hemoglobin that may be present in erythrocytes when patients have glucose-6-phosphate dehydrogenase deficiency or thalassemia. They cannot be seen on routine staining. (Choice D) Spherocytes have low surface area-to-volume ratios and are more susceptible to osmotic stress. They are seen most commonly in hemolytic anemia and hereditary spherocytosis and appear as dense, round red blood cells. Target cells are less susceptible to osmotic stress than normal red blood cells or spherocytes due to their excess membrane. (Choice E) Basophilic stippling is blue granules dispersed within the cytosol of erythrocytes due to precipitated ribosomes. It is seen most commonly in thalassemias, alcohol use disorder, and lead/heavy metal poisoning. Educational objective: Target cells form when erythrocytes have reduced cell volume (eg, thalassemia, iron deficiency) or excessive membrane (eg, obstructive liver disease, postsplenectomy). Patients who undergo splenectomy usually develop target cells because the spleen is the primary organ that prunes excessive red cell membrane.
A 16-year-old girl comes to the office due to fatigue for the past few months. She attends high school and plays on the school soccer team but says that her endurance has decreased. The patient sleeps 9 hours a night and has been a vegetarian for the past 6 months. She reached menarche at age 13 and has had regular menses for the past 6 months. Blood pressure is 110/60 mm Hg, pulse is 70/min, and BMI is 21 kg/m2. Physical examination shows a well-nourished teenage girl with pale conjunctivae. Hemoglobin is 9.2 g/dL. Which of the following sets of additional laboratory findings are most likely to be seen in this patient? Serum ferritin, circulating transferring, MCV, hypersegmented neutrophils, serum folate normal, normal, 74, none, normal low, high, 76, none, normal low, low 84, none, normal high, low, 95, none, normal normal, normal, 108, present, normal normal, normal, 115, present, low
This girl with fatigue, conjunctival pallor, and a low hemoglobin level has anemia. Women of childbearing age are at risk for iron-deficiency anemia due to menstrual cycle blood loss, especially teenage girls who have higher iron requirements due to growth. In this patient, decreased consumption of dietary iron (eg, vegetarianism) is an additional risk factor. As iron deficiency develops, the following sequence can be seen: Decreased bone marrow iron stores (ferritin and hemosiderin) Decreased serum ferritin Increased serum total iron-binding capacity, reflecting increased transferrin (Choice C) Decreased serum iron concentration Decreased hemoglobin Appearance of microcytic, hypochromic red blood cells (low mean corpuscular volume [MCV]) Ferritin is an intracellular iron-storage protein that is used as a serum marker of total body iron stores. It is decreased in iron deficiency and elevated in iron overload or during infection/inflammation (acute phase reaction). Transferrin transports iron through the plasma. When iron levels are normal, approximately one third of circulating transferrin is bound to iron. In iron deficiency, hepatic synthesis of transferrin increases but transferrin saturation drops due to decreased release of iron into the plasma from intracellular stores. (Choice A) The earliest signs of iron deficiency are low serum ferritin and high serum transferrin, which manifest before overt anemia develops. This pattern of findings is more consistent with α- or β-thalassemia, which usually cause microcytic anemia with normal ferritin and transferrin levels. (Choice D) Low hemoglobin in the setting of a normal MCV, low circulating transferrin, and high serum ferritin is suggestive of anemia of chronic disease, which is often associated with infections and inflammatory conditions. Other acute phase reactants (eg, C-reactive protein, sedimentation rate) are usually elevated in these patients. (Choices E and F) Hypersegmented neutrophils are characteristic of megaloblastic anemias, which can be caused by folate and vitamin B12 deficiency. Folate deficiency is unlikely in this patient, as folate is found in many vegetable products. Although vegetarians are at risk for vitamin B12 deficiency, depletion of hepatic B12 stores takes several years, and this patient has been a vegetarian only for the past 6 months. Educational objective: Anemia in women of childbearing age is typically caused by iron deficiency secondary to menstrual blood loss. Iron deficiency is associated with decreased serum ferritin, increased total iron-binding capacity (transferrin), and microcytic, hypochromic red blood cells.
A 54-year-old man comes to the emergency department with pain and swelling in the right leg. The patient has no other medical problems and takes no medications. He has smoked a pack of cigarettes daily for the past 30 years. He is a business executive and just returned from an overseas trip. Physical examination shows pitting edema of the right leg and tenderness on deep palpation of the calf muscles. Peripheral pulses are symmetric and full. He is started on a medication that prolongs activated partial thromboplastin time and prothrombin time in a dose-dependent manner but has no effect on thrombin time. Which of the following agents was this patient most likely administered? Cyclooxygenase inhibitor Direct factor VIIa inhibitor Direct factor Xa inhibitor Direct thrombin inhibitor Unfractionated heparin
This patient has deep venous thrombosis, which is typically treated with anticoagulation therapy for >3 months to prevent recurrent thrombosis. Initial treatment is often with heparin agents (eg, unfractionated heparin, low-molecular-weight heparin, fondaparinux), followed by bridging to an oral anticoagulant (most commonly warfarin). Additional treatment options include direct factor Xa inhibitors (eg, apixaban, rivaroxaban) and direct thrombin inhibitors (eg, dabigatran). In general, medications that affect the extrinsic pathway prolong the prothrombin time (PT), whereas inhibition of the intrinsic pathway mainly prolongs the activated partial thromboplastin time (aPTT). The thrombin time (TT) is prolonged with medications that directly or indirectly inhibit thrombin. Because factor Xa is located at the junction of the intrinsic and extrinsic pathways, direct factor Xa inhibitors prolong both aPTT and PT with no effect on TT. (Choice A) Cyclooxygenase inhibitors (eg, aspirin) directly inhibit platelet activity and have little overall effect on aPTT, PT, or TT. (Choice B) Direct factor VIIa inhibitors are currently in development and work by inhibiting the extrinsic pathway of the coagulation cascade. Therefore, they would prolong PT with no effect on aPTT or TT. (Choice D) Direct thrombin inhibitors (eg, dabigatran) prevent the formation of thrombin (factor IIa), resulting in prolongation of aPTT, PT, and TT. (Choice E) Unfractionated heparin binds to antithrombin and causes inactivation of several coagulation factors, most significantly thrombin and factor Xa. As a result, aPTT and TT will be prolonged. Although the PT should theoretically be prolonged by the action of heparin on the final common pathway, in practice, the PT reagent contains heparin neutralizers that minimize this effect. Educational objective: Direct factor Xa inhibitors (eg, apixaban, rivaroxaban) increase the prothrombin and activated partial thromboplastin times but do not affect the thrombin time. Unfractionated heparin and direct thrombin inhibitors (eg, dabigatran) prolong the thrombin time.
A 20-year-old woman comes to the emergency department due to bloody stools. Approximately an hour ago, the patient had a bowel movement that appeared grossly bloody. For the past month, she has also had decreased energy. Temperature is 37 C (98.6 F), blood pressure is 110/60 mm Hg, pulse is 110/min, and respirations are 20/min. The patient appears tired. Cardiopulmonary examination reveals mild tachycardia. The abdomen is soft without organomegaly. Skin examination shows pallor and scattered bruises in various stages of healing throughout the trunk. Complete blood count results are as follows: Hemoglobin 7.2 g/dL Mean corpuscular volume 90 µm3 Platelets 10,000/mm3 Leukocytes 1,050/mm3 Neutrophils 5% Lymphocytes 95% Which of the following is the most likely cause of this patient's condition? Autoimmune-induced loss of self-renewing hematopoietic stem cells Bone marrow replacement by constitutively active tyrosine kinase-stimulated cells Splenic hyperactivity with cell trapping by the reticuloendothelial system Increased destruction of cells opsonized by glycoprotein IIb/IIIa targeted autoantibodies Ineffective hematopoiesis secondary to micronutrient deficiency
This patient has gastrointestinal bleeding and ecchymosis secondary to thrombocytopenia; although anemia (eg, pallor) would be expected with significant bleeding, her low leukocyte count indicates pancytopenia, which is generally caused by one of the following: Bone marrow aplasia: Hematopoietic stem cells in the bone marrow are unable to proliferate and differentiate into mature blood cells due to aplastic anemia, nutritional impairment, infection (eg, parvovirus, HIV), or cytotoxic medications. Bone marrow infiltration: Cancer, fibrosis, or infection (eg, tuberculosis) fills the bone marrow and crowds out hematologic cells, thereby preventing blood cell replication and maturation. Mature blood cell destruction: Circulating mature blood cells may be destroyed due to disseminated intravascular coagulation or thrombotic thrombocytopenic purpura; extravascular mature blood cells may be destroyed or sequestered in the spleen due to hypersplenism. Based on this patient's laboratory and physical examination results, several answer choices can be eliminated. The presence of a normal MCV makes vitamin B12 or folate deficiency unlikely because these are associated with macrocytic (not normocytic) anemia (Choice E). Hypersplenism is also unlikely because abdominal examination would show significant splenomegaly (Choice C). Chronic myelogenous leukemia, an infiltrative cancer associated with constitutively active tyrosine kinase, is also unlikely because this condition is usually associated with dramatic leukocytosis (not leukopenia) (Choice B). Therefore, the most likely cause of this patient's pancytopenia is aplastic anemia (AA). Although AA is linked to certain medications, infections, toxins, or radiation, most cases are idiopathic and thought to be caused by autoimmune-induced loss of multipotent hematologic stem cells. In idiopathic AA, an underlying insult (eg, mutation, virus) causes alteration of surface antigens on multipotent stem cells, making them appear foreign and triggering a cytotoxic T-cell response. Cytokines released by T-helper cells (type 1 cytokines) also contribute to the pathogenesis, most notably interferon-gamma, which triggers apoptotic cell death due to the stimulation of a destructive cytokine cascade and the increased expression of the Fas receptor on the hematologic stem cell surface. (Choice D) Autoantibodies against glycoprotein IIb/IIIa cause immune thrombocytopenia, which is marked by thrombocytopenia and no change in the other cell lines. Educational objective: Aplastic anemia is a form of bone marrow failure in which patients have decreased production of all cell lines (platelets, erythrocytes, and leukocytes). It is primarily caused by the autoimmune destruction of multipotent hematologic stem cells due to an alteration in their surface antigens, leading to a cytotoxic T-cell response and release of interferon-gamma from T-helper cells (triggers apoptotic cell death).
A 24-year-old woman is being evaluated for chronic fatigue. She has a history of heavy menstrual periods since menarche and also recalls frequent nosebleeds as a child. Her past medical history is otherwise insignificant and she takes no medications. Laboratory studies show a hemoglobin level of 9.2 g/dL, a mean corpuscular volume of 72 fL, and decreased levels of ferritin. Item 1 of 2 Which of the following is the most likely diagnosis? Antiphospholipid syndrome Factor VIII deficiency Factor XIII deficiency Immune thrombocytopenia Protein C deficiency von Willebrand factor deficiency
This patient has microcytic anemia with a low ferritin level, which is characteristic of iron deficiency anemia. In developed countries, iron deficiency anemia is most commonly caused by overt or occult blood loss. The patient's history of heavy menstrual periods (menorrhagia) since menarche and frequent nosebleeds (epistaxis) as a child further suggests von Willebrand disease (vWD) as the most likely etiology of her anemia. In vWD, a mutation results in impaired synthesis in von Willebrand factor, leading to a qualitative defect in platelet binding and aggregation. As a result, patients with vWD experience easy bleeding from skin and mucosal sites, including the gingivae, nasal mucosa, gastrointestinal tract, and endometrium. (Choice A) Antiphospholipid syndrome causes a hypercoagulable state that increases the risk of thromboses and spontaneous abortion. (Choice B) Factor VIII deficiency causes hemophilia A. Severe forms of vWD can also result in factor VIII deficiency as factor VIII is protected from degradation by vWF. However, this patient does not have the characteristic deep-tissue bleeding (eg, bleeding into joints/muscles, gastrointestinal bleeding, hematuria) typically seen with clotting factor deficiencies. (Choice C) Factor XIII deficiency is an extremely rare autosomal recessive disorder that causes clot instability. Affected patients often have delayed, recurrent bleeding after trauma or surgery. Hemophilia-like bleeding (eg, deep-tissue bleeding) is seen in factor XIII deficiency. (Choice D) Immune thrombocytopenia (formerly immune thrombocytopenic purpura) produces isolated thrombocytopenia with episodic bleeding that is typically mucocutaneous (eg, petechiae, purpura, epistaxis). However, the associated bleeding is unlikely to be chronic and unrelenting since childhood. (Choice E) Protein C is a natural anticoagulant protein, and deficiency results in a procoagulant state that most commonly manifests with recurrent deep venous thromboses. Educational objective: Patients with von Willebrand disease often present with a lifelong history of mucosal bleeding, including gingival bleeding, epistaxis, and/or menorrhagia. These patients have normal platelet levels but typically have a prolonged bleeding time due to impaired platelet functioning.
A 70-year-old man comes to the office due to palpitations. He has felt an irregular and fast heartbeat on multiple occasions over the last month. The patient has not had chest pain, dyspnea, dizziness, or weakness. His medical history includes hypertension and type 2 diabetes mellitus. Temperature is 36.7 C (98.1 F), pulse is 102/min and irregular, blood pressure is 140/82 mm Hg, and respirations are 16/min. Cardiopulmonary auscultation reveals clear lungs and no heart murmurs. ECG shows an irregularly irregular rhythm with absent P waves. The patient is started on metoprolol and apixaban. Which of the following is likely to be directly affected as the result of this therapy? Conversion of factor X to Xa Conversion of fibrinogen to fibrin Conversion of plasminogen to plasmin Conversion of prothrombin to thrombin Gamma-carboxylation of coagulation factors
This patient has palpitations, an irregular pulse, and ECG findings consistent with atrial fibrillation. Treatment with a rate control medication (eg, metoprolol) reduces atrioventricular conduction and limits ventricular tachycardia, which helps prevent symptoms (eg, palpitations, dizziness) and risk of cardiomyopathy. However, rate control agents do not inhibit uncoordinated atrial contraction, so patients remain at risk for atrial thrombus and embolic stroke. As a result, most patients are also initiated on anticoagulation. Direct factor Xa inhibitors (eg, apixaban) are often used for stroke prevention in atrial fibrillation as they are administered orally and do not require monitoring of drug levels (unlike warfarin). This class of medications blocks the active site of factor Xa, which prevents it from converting prothrombin to thrombin. Direct factor Xa inhibitors are denoted by names that end in "Xa-ban." (Choice A) The conversion of factor X to factor Xa is the beginning the final common pathway of the coagulation cascade and is triggered by factor VIIIa (intrinsic pathway) and tissue factor (extrinsic pathway). Factor Xa inhibitors block conversion of prothrombin to thrombin, not factor X to factor Xa. (Choice B) Factor Xa inhibitors reduce the conversion of fibrinogen to fibrin. However, this is a downstream (not direct) effect that occurs as a result of decreased thrombin levels. (Choice C) Tissue plasminogen activator is a fibrinolytic enzyme that increases the conversion of plasminogen to plasmin. It is primarily used to lyse ongoing clots in patients with acute ischemic strokes. (Choice E) Warfarin inhibits vitamin K epoxide reductase, which decreases the reduced form of vitamin K and limits gamma-carboxylation of vitamin K-dependent coagulation factors (II, VII, IX, X). Educational objective: Direct factor Xa inhibitors (eg, apixaban) are anticoagulants that block the active site of factor Xa, which leads to reduced conversion of prothrombin to thrombin. This class of medications is administered orally and requires no drug level monitoring.
A 61-year-old man comes to the hospital due to 2 days of right leg swelling. He recently flew back to the United States from Japan, where he was visiting his daughter who is spending a semester abroad. The patient has a medical history of hypertension and hyperlipidemia. Temperature is 37.2 C (99 F), blood pressure is 136/70 mm Hg, and pulse is 90/min. Pulse oximetry shows 99% on room air. Cardiopulmonary examination is within normal limits. There is tenderness and swelling of the right lower extremity up to the mid thigh. Laboratory studies, including prothrombin time and activated partial thromboplastin time, are normal. Venous doppler reveals an occluding thrombus in the right femoral and popliteal veins. The patient is started on rivaroxaban. This drug inhibits which of the following? Factor VIII Factor Xa Glycoprotein IIb/IIIa Platelet P2Y12 Thrombin
This patient has right lower extremity proximal deep venous thrombosis (DVT), with his recent long flight from Japan being a possible risk factor. Many patients with DVT receive long-term anticoagulation with warfarin. Alternates include direct factor Xa inhibitors (eg, rivaroXaban, apiXaban), which bind to the active site of factor Xa and prevent thrombin formation. These drugs can be administered orally as monotherapy and do not require laboratory monitoring. They are used for venous thromboembolism treatment as well as for stroke prophylaxis in patients with atrial fibrillation (who are at risk for embolic stroke). (Choice A) Factor VIII replacement (not inhibition) is the mainstay of hemophilia A treatment. (Choice C) Eptifibatide inhibits platelet aggregation and thrombosis by blocking the glycoprotein IIb/IIIa receptor, which is the binding site for fibrinogen. This drug is typically used in acute coronary syndrome and in patients undergoing percutaneous coronary intervention. (Choice D) Ticagrelor binds the adenosine diphosphate (ADP) P2Y12 receptor on platelets, which prevents platelet aggregation by blocking ADP-mediated activation of the glycoprotein IIb/IIIa receptor complex. This drug is also used in acute coronary syndrome and in patients undergoing percutaneous coronary intervention. (Choice E) Dabigatran is an oral direct thrombin inhibitor that inactivates both circulating and clot-associated thrombin. It is also used for treatment of thromboembolic disease and stroke prophylaxis in atrial fibrillation. Educational objective: Rivaroxaban is an oral anticoagulant that directly inhibits factor Xa. It is used in venous thromboembolism and atrial fibrillation.
A 29-year-old woman comes to the emergency department due to fever and headache for the last week. The patient has a generalized tonic-clonic seizure while being evaluated. Laboratory results are as follows: Hemoglobin 6.1 g/dL Platelets 16,000/mm3 Creatinine 2.2 mg/dL Total bilirubin 4.3 mg/dL Serum haptoglobin undetectable PT 11 sec (INR 1.1) Activated PTT 30 sec Peripheral blood smear is shown in the exhibit (helmet cells). Which of the following is the most likely underlying cause of this patient's current condition? Bacterial product-mediated overaction of coagulation cascade Decreased activity of von Willebrand factor-cleaving protease Expansion of neoplastic myeloid precursor cells in bone marrow Inherited deficiency of glucose-6-phosphate dehydrogenase enzyme Missense mutation at the sixth position of the hemoglobin beta chain
This patient has the classic pentad of manifestations for acquired thrombotic thrombocytopenic purpura (TTP): Severe thrombocytopenia: Platelet-rich clots form in the microvasculature, leading to rapid platelet consumption. Microangiopathic hemolytic anemia (MAHA): Erythrocytes are mechanically sheared by the microvascular thrombi, leading to intravascular hemolytic anemia. This typically causes an elevated indirect bilirubin level and an undetectable haptoglobin level (haptoglobin binds free hemoglobin in the circulation), but confirmation of MAHA requires identification of schistocytes (eg, triangle cells, helmet cells) on peripheral blood smear. Neurologic manifestations (eg, headache, seizure), renal insufficiency, and fever (the remainder of the pentad) occur in a minority of patients Unlike patients who have disseminated intravascular coagulation (a consumptive coagulopathy also associated with MAHA and thrombocytopenia), those with TTP have normal coagulation studies because coagulation factors are not significantly consumed by the formation of platelet-rich clots (Choice A). TTP is triggered by the formation of autoantibody inhibitors against ADAMTS-13, a protease that cleaves ultralarge von Willebrand factor (VWF) multimers in the circulation, reducing their prothrombotic activity. Patients with TTP have very low levels of ADAMTS-13, which increases the proportion of ultralarge VWF multimers and leads to the aggregation and activation of platelets. Plasma exchange, which removes the autoantibody inhibitors against ADAMTS-13, is generally curative. (Choice C) Expansion of neoplastic myeloid precursors in the bone marrow is seen in acute myeloid leukemia, which can cause anemia and thrombocytopenia due to bone marrow infiltration. However, peripheral blood smear would show leukemic blast cells, not schistocytes. (Choice D) Inherited deficiency of glucose-6-phosphate dehydrogenase typically causes hemolytic anemia during times of oxidative stress (eg, infection, medication exposure, certain foods). High bilirubin and low haptoglobin are often present due to hemolytic anemia; however, thrombocytopenia is not seen, and peripheral smear would show bite cells and Heinz bodies, not schistocytes. (Choice E) A missense mutation at the sixth position of the hemoglobin beta chain is seen in sickle cell anemia, which is marked by periods of acute on chronic hemolytic anemia (eg, elevated bilirubin, low haptoglobin). However, peripheral smear would show sickled erythrocytes, not schistocytes; in addition, thrombocytopenia is not generally seen. Educational objective: Thrombotic thrombocytopenic purpura classically presents with the pentad of severe thrombocytopenia, microangiopathic hemolytic anemia (eg, schistocytes on peripheral smear), renal insufficiency, neurologic symptoms, and fever. However, all these signs and symptoms are rarely present. Diagnosis is often made by identifying severe deficiency of ADAMTS-13, a protease that cleaves large von Willebrand factor multimers off the endothelium.
A 12-year-old boy has prolonged oral bleeding immediately after a tooth extraction. Despite several interventions, the bleeding persists for hours and stops only after desmopressin (DDAVP) administration. The patient has several dental caries due to excessive juice intake and inconsistent oral hygiene. He has no other medical problems and takes no medications. His father and paternal grandmother have also had excessive bleeding after dental procedures. Review of systems is positive for mild bruising on his legs. Which of the following is the most likely therapeutic mechanism of desmopressin for this patient's condition? Binding to renal tubular cell receptors Increase in endothelial protein release Increase in liver protein synthesis Inhibition of fibrinolysis Vascular smooth muscle contraction
This patient likely has von Willebrand disease (vWD), an autosomal dominant deficiency of von Willebrand factor (vWF). vWF is secreted by endothelial cells and circulates in multimers that are non-covalently attached to clotting factor VIII, increasing its stability. vWF is also responsible for augmenting platelet binding at endothelial injury sites, which helps in the formation of the initial platelet plug. Patients have increased bruisability and mucosal bleeding (eg, oropharyngeal, gastrointestinal, uterine). Common manifestations include prolonged bleeding during dental procedures and heavy menstrual periods. Most patients do not require medical attention as vWD symptoms are typically mild. However, those who need treatment can receive desmopressin (or 1-desamino-8-d-arginine vasopressin [DDAVP]), a synthetic analog of vasopressin (ie, antidiuretic hormone [ADH]), which is normally released by the posterior pituitary. DDAVP increases vWF release from endothelial cells. DDAVP also raises factor VIII levels and is used for the treatment of mild hemophilia A. Hemophilia is an X-linked coagulation disorder and is unlikely in this patient given the clear autosomal dominant pattern of inheritance (father and paternal grandmother). (Choice A) DDAVP also acts as an ADH analogue by binding to V2 receptors in the renal tubular cells, which leads to increased aquaporin channels, increased water reabsorption, and decreased urine output. Due to this action, DDAVP can be used in the treatment of central diabetes insipidus and nocturnal enuresis. (Choice C) Although most clotting factors are synthesized in the liver, vWF and factor VIII are produced in endothelial cells. DDAVP has no known effect on liver protein synthesis. (Choice D) Inhibition of fibrinolysis (breaking down fibrin in blood clots) can decrease the rate of clot breakdown and potentially slow heavy bleeding. Aminocaproic acid is an anti-fibrinolytic medication used to treat excessive postoperative bleeding. DDAVP has no effect on the fibrinolytic system. (Choice E) Vascular smooth muscle cells are involved in blood pressure regulation, with contraction leading to higher blood pressure. Endogenous vasopressin acts via the V1 vasopressin receptor to raise blood pressure through vascular smooth muscle contraction. Although DDAVP is chemically similar to vasopressin, it has a minimal effect on the V1 vasopressin receptor. Educational objective: The von Willebrand factor (vWF) enhances clotting through both augmentation of platelet binding and stabilization of factor VIII. Patients with von Willebrand disease are deficient in functional vWF and present with increased bruisability and prolonged mucosal bleeding. Desmopressin can alleviate bleeding through endothelial release of vWF.
A 48-year-old woman comes to the emergency department with headache, dizziness, and nausea for the past several hours. She has no fever, nasal congestion, or cough but reports that her husband has also been complaining of headache. The patient has been burning wood in the fireplace to warm her house after losing electricity during a snowstorm. Her temperature is 37 C (98.6 F), blood pressure is 135/70 mm Hg, and pulse is 94/min and regular. Physical examination is unremarkable. The substance responsible for this patient's condition most likely impairs hemoglobin function through which of the following mechanisms? Alteration of the partial pressure of oxygen Competitive binding to heme Deactivation of a reductase enzyme Denaturation of the globin chains Irreversible linking to heme Oxidation of the porphyrin ring
This patient most likely has carbon monoxide (CO) poisoning, a condition that presents with nonspecific findings ranging from headache and dizziness to convulsions and respiratory arrest depending on concentration and exposure. CO is a colorless, odorless, tasteless gas that is a byproduct of incomplete hydrocarbon combustion. CO poisoning is most often caused by smoke inhalation from a fire or the burning of fuel sources such as wood, coal, or natural gas in poorly ventilated environments. CO toxicity occurs because of CO's ability to competitively bind iron present in heme proteins. The gas binds to heme iron with a much higher affinity than oxygen, forming carboxyhemoglobin. Even if only 1 of the 4 heme sites is affected, the remaining 3 heme groups have increased oxygen affinity (leftward shift of the oxygen dissociation curve), impeding oxygen delivery to tissues. CO is also capable of binding cardiac myoglobin with high affinity, disrupting the heart's ability to use oxygen and thereby decreasing cardiac output. At a cellular level, CO binds to cytochrome oxidase, inhibiting aerobic metabolism and exacerbating tissue hypoxia. Treatment for CO poisoning is administration of high-flow or hyperbaric oxygen therapy as this hastens the dissociation of CO from carboxyhemoglobin. (Choice A) The partial pressure of oxygen (pO2) is a measure of dissolved oxygen in the plasma, the amount of which does not change in CO poisoning. (Choice C) NADH methemoglobin reductase reduces ferric iron (Fe3+) to ferrous iron (Fe2+), regenerating hemoglobin from methemoglobin. Enzyme deficiency results in congenital methemoglobinemia. (Choice D) Globin chain denaturation occurs in G6PD-deficient red blood cells when oxidant stressors cause sulfhydryl group cross-linking. (Choice E) CO reversibly binds hemoglobin at its heme moieties. (Choice F) Heme consists of an iron bound to protoporphyrin IX. CO binds to iron in metalloproteins but does not cause oxidation of the porphyrin component. Educational objective: Carbon monoxide binds heme iron in hemoglobin with an affinity much greater than oxygen, generating carboxyhemoglobin. Remaining binding sites on carboxyhemoglobin have an increased affinity for oxygen that causes the oxygen dissociation curve to shift to the left, impeding oxygen delivery to tissues.
A 66-year-old woman is brought to the emergency department due to left leg pain. Five days ago, the patient underwent elective right hip arthroplasty for advanced osteoarthritis. She had been recovering well at an acute rehabilitation facility until she woke today with severe left calf pain and swelling. The patient has taken daily subcutaneous, low-molecular-weight heparin and as-needed acetaminophen since the surgery. She has no other medical conditions. Temperature is 37 C (98.6 F), blood pressure is 130/70 mm Hg, and pulse is 92/min. The surgical incision over her right hip is healing well. Her left posterior calf is tender and swollen to the knee. Three days ago, complete blood count was normal, but platelet count is now 70,000/mm3. Peripheral blood smear shows decreased platelet number but no other abnormalities. Which of the following is the most likely cause of this patient's thrombocytopenia? Decreased bone marrow platelet production Mechanical destruction of platelets Non-immune mediated platelet clumping Platelet removal by splenic macrophages Uncontrolled activation of the coagulation cascade
This patient on low-molecular-weight heparin developed thrombocytopenia and deep venous thrombosis (calf pain/swelling), raising strong suspicion for heparin-induced thrombocytopenia (HIT) type 2. In HIT type 2, heparin induces a conformational change in platelet-factor 4, leading to the formation of a neoantigen. IgG antibodies form against the neoantigen (heparin-platelet factor 4), which results in antibody aggregation on the platelet surface. Patients generally develop manifestations 5-10 days after heparin initiation. Thrombocytopenia is the hallmark feature due to the clearance of antibody-coated platelets by splenic macrophages. Antibody aggregation on the platelet surface also results in wide-spread platelet aggregation, which worsens the thrombocytopenia and significantly increases the risk of venous and arterial thrombosis. Patients with HIT-type 2 should immediately stop heparin and be initiated on a nonheparin anticoagulant (eg, argatroban) to help prevent/treat thrombosis. (Choice A) Although bone marrow suppression can cause thrombocytopenia, it most often occurs in the setting of malignant invasion of the bone marrow (eg, leukemia) or chemical toxicity (eg, chemotherapy, excessive alcohol). Thrombosis is uncommon. (Choices B and E) Mechanical destruction of platelets can be seen with a mechanical prosthetic heart valve or in disseminated intravascular coagulation, a condition marked by the excessive activation of the coagulation cascade. In both of these conditions, thrombocytopenia is accompanied by schistocytes on peripheral blood smear. (Choice C) HIT type 1 is a non-immune-mediated condition caused by platelet clumping. Thrombocytopenia is typically mild (nadir ~100,000/mm3), and most cases arise within 2 days of heparin administration. It does not cause thrombosis and is not considered clinically significant; therefore, heparin can be continued. Educational objective: Heparin-induced thrombocytopenia type 2 results from the formation of IgG antibodies to complexes of heparin and endogenous platelet factor 4. The major manifestation is thrombocytopenia, which typically occurs 5-10 days after heparin initiation due to the destruction of antibody-coated platelets by splenic macrophages. Widespread intravascular platelet activation also results in a high risk of arterial and venous thrombosis.
A 50-year-old woman comes to the emergency department due to a large, painful skin lesion. She does not remember sustaining any trauma. She was recently diagnosed with atrial fibrillation and started on treatment with warfarin. The patient noticed the lesion about a day and a half after starting the medication. Temperature is 36.7 C (98 F), blood pressure is 130/82 mm Hg, and pulse is 88/min and irregularly irregular. Physical examination findings are shown in the image below. ID=1292 The remainder of the skin examination is normal. Which of the following is the most likely cause of this patient's skin lesion? Allergic-drug reaction Antithrombin deficiency Autoimmune phenomena Protein C deficiency Vitamin K deficiency
This patient was started on warfarin to prevent thromboembolism secondary to atrial fibrillation. The sharply demarcated purpuric lesion likely represents warfarin-induced skin necrosis, a rare complication that results from an excessive decrease in protein C activity; patients with underlying protein C deficiency are at increased risk. Protein C and S are innate anticoagulants that are vitamin K dependent, as are procoagulant factors II, VII, IX, and X. Warfarin inhibits epoxide reductase (responsible for regenerating vitamin K), which leads to decreased levels of these proteins. Factor VII and protein C are depleted first due to their shorter half-life. The early loss of protein C activity relative to procoagulant factor activity leads to a transient hypercoagulable state that persists until the other procoagulant factors are sufficiently inhibited. In patients with protein C deficiency, this procoagulant state is exaggerated, promoting microvasculature occlusion and hemorrhagic skin necrosis. Underlying protein S deficiency also increases the risk of warfarin-induced skin necrosis, though not to the same extent. Treatment of warfarin-induced skin necrosis includes discontinuing warfarin and administering fresh frozen plasma or protein C concentrate. The condition may be prevented by the coadministration of heparin during the first several days of warfarin therapy. (Choice A) Allergic drug reactions are immune-mediated hypersensitivity reactions that can have variable cutaneous findings ranging from diffuse erythematous eruptions to extensive blistering and exfoliation (eg, Stevens-Johnson syndrome/toxic epidermal necrolysis); this patient's recent warfarin use and localized skin findings are more suggestive of warfarin-induced skin necrosis. (Choice B) Antithrombin deficiency can be inherited (autosomal dominant) or acquired (eg, cirrhosis, nephrotic syndrome) and presents with venous thromboembolism (eg, deep vein thrombosis, pulmonary embolism) and resistance to heparin. Warfarin does not affect antithrombin activity. (Choice C) Heparin-induced thrombocytopenia is an autoimmune phenomenon that causes arterial and venous thrombosis. It can produce skin findings similar to this patient's but would generally be associated with thrombocytopenia and recent heparin use. (Choice E) Vitamin K deficiency most often occurs in patients with fat malabsorption or those taking broad-spectrum antibiotics. The deficiency increases the inhibitory effect of warfarin on all vitamin K-dependent proteins (both anticoagulant and procoagulant), so it does not alter the relative coagulability balance to encourage warfarin-induced skin necrosis. Education objective: Patients started on warfarin develop a transient hypercoagulable state due to the short half-life of protein C. This hypercoagulability is further exaggerated by preexisting protein C deficiency and can result in thrombotic occlusion of the microvasculature with skin necrosis.
ThromboticA 5-year-old girl is brought to the emergency department for evaluation of a rash. Her mother noticed a few red spots on the child's cheeks this morning, which subsequently spread to the neck and trunk as the day progressed. The patient has a history of partial complex seizures, which have been well controlled with carbamazepine. Vital signs are normal. The lips are slightly pale, and the oropharynx is clear. There is no lymphadenopathy. The abdomen is soft with no palpable masses or hepatosplenomegaly. Skin examination shows flat, red, pinpoint lesions that do not blanch on the face, neck, trunk, and extremities. Neurological examination is unremarkable. Laboratory results are as follows: Complete blood count Hemoglobin 9.5 g/dL Platelets 25,000/mm3 Leukocytes 2,900/mm3 Peripheral smear reveals a paucity of cells with normal morphologic appearance of all cell lines. Which of the following mechanisms is most likely responsible for this patient's current condition? Autoantibodies against hematopoietic stem cells Impaired DNA synthesis due to vitamin deficiency Medication associated adverse effect Peripheral cell sequestration in splenic sinusoids Thrombtic microangiopathic hemolysis
This patient who takes carbamazepine developed pancytopenia with morphologically normal peripheral blood cells, indicating likely drug-induced aplastic anemia (AA). AA is a form of bone marrow failure caused by damage to multipotent hematopoietic stem cells. Most cases are induced by direct toxicity to hematopoietic stem cells or an alteration to stem cell surface antigens, which leads to their subsequent destruction by cytotoxic lymphocytes. Although AA can be idiopathic, it is often triggered by ionizing radiation, toxic chemicals, viral infections, or medications. Drug-induced AA is typically categorized as follows: Dose-dependent reactions are caused by medications (eg, chemotherapy, immunosuppressives) that are toxic to the bone marrow when drug levels exceed a certain concentration for a certain period. They tend to be predictable in onset. Idiosyncratic reactions are often caused by antiseizure medications (eg, carbamazepine, valproic acid), sulfonamides, or nifedipine. They are unpredictable, unrelated to the dose of the drug, and may occur at any point during or after therapy. Although the mechanism is unclear, patients often have genetic mutations in drug-metabolizing enzymes or efflux pumps, which may promote toxicity. Because multipotent hematopoietic cells generate all mature blood cells, patients with AA usually have signs of thrombocytopenia (eg, petechiae, bleeding), anemia (eg, pale mucous membranes), and/or leukopenia (eg, infections). Bone marrow biopsy will reveal a hypocellular marrow with increased lipids. (Choice A) AA is primarily caused by toxic injury to hematopoietic stem cells or cytotoxic T-cell-mediated stem cell destruction; autoantibodies are not a major part of the pathogenesis. (Choice B) Vitamin B12 and folate deficiencies impair DNA synthesis in hematopoietic stem cells, which leads to hypersegmented neutrophils and macrocytic erythrocytes (not normal morphologic cells) on peripheral smear because of delayed nuclear (relative to cytoplasmic) maturation. (Choice D) Hypersplenism can cause sequestration with increased destruction of blood cells in the splenic sinusoids. Most cases arise in those with portal hypertension (eg, liver disease, portal vein thrombosis), which leads to an increase in splenic volume and activity. Although cytopenias can occur, they are typically mild and do not cause symptoms; in addition, most patients have splenomegaly. (Choice E) Thrombotic microangiopathic hemolysis is associated with clot formation in the microvasculature and shearing of red blood cells, leading to thrombocytopenia (eg, petechiae) and anemia. However, peripheral blood smear would reveal schistocytes. Educational objective: Aplastic anemia (AA) is bone marrow failure due to multipotent hematopoietic stem cell destruction. Cases are often triggered by medications, radiation, toxins, or viruses. Medication-induced AA can be due to dose-dependent bone marrow damage (eg, chemotherapy, immunosuppressive drugs) or idiosyncratic reactions (eg, antiseizure medicines, sulfonamides, nifedipine) that may occur at any point during or after therapy.
A 26-year-old woman, gravida 2 para 1, at 8 weeks gestation comes to the office due to pain and swelling of her left leg for the past day. The patient had a pulmonary embolism during her previous pregnancy, and prophylactic low-molecular-weight heparin therapy was initiated 6 days ago. She has no other medical conditions and takes prenatal vitamins. Physical examination shows left lower extremity edema and calf tenderness but no other abnormalities. Venous duplex ultrasonography reveals acute left femoral vein thrombosis. Platelet count, which was normal prior to anticoagulant therapy initiation, is 84,000/mm3. Other blood cell counts and renal and liver function studies are within normal limits. Which of the following most likely predisposed this patient to her current condition? Acquired protein C deficiency Anti-platelet factor 4/heparin antibodies Anti-platelet glycoprotein IIb/IIIa antibodies Cold-precipitate immunoglobulins Decreased ADAMTS13 levels
This patient with a significant drop in platelet count and acute venous thromboembolism following recent exposure to low molecular weight heparin likely has heparin-induced thrombocytopenia and thrombosis (HITT), also known as heparin-induced thrombocytopenia type 2. HITT typically occurs 5-10 days following exposure to heparin products and is characterized by a large drop in platelet count. It is more common following exposure to unfractionated heparin but can occur following exposure to low-molecular weight heparin as well. Platelet factor 4 (PF4) is a protein released from the alpha granules of platelets that plays a role in platelet aggregation. It also binds heparin and helps inactivate the molecule. The mechanism of HITT involves the generation of IgG antibodies to these complexes of heparin and PF4. The Fc component of the activated IgG antibodies then binds to additional platelets, resulting in further PF4 release and widespread platelet activation. This leads to a prothrombotic state that places patients at high risk for both arterial and venous thrombosis. (Choice A) Acquired protein C deficiency occurs early in the course of warfarin therapy, as the inhibition of protein C by warfarin occurs more rapidly than the inhibition of other factors (ie, factors II, VII, IX, X). If not bridged with heparin when starting therapy, patients may develop warfarin-induced skin necrosis due to localized cutaneous thrombus formation. (Choice C) Idiopathic thrombocytopenic purpura (ITP) results from splenic destruction of platelets labeled by IgG antibodies to glycoprotein IIb/IIIa receptors. ITP often causes very low platelet levels and is associated with bleeding complications rather than thrombosis. (Choice D) Cryoglobulinemia can occur in the setting of autoimmune disease (eg, systemic lupus erythematosus) or viral infection (eg, hepatitis C) and causes systemic vasculitis characterized by fatigue, arthralgia, and purpuric rash. Marked thrombocytopenia is not typical. (Choice E) Thrombotic thrombocytopenic purpura (TTP) results from decreased levels of the von Willebrand factor-cleaving protease ADAMTS13. The classic presentation of TTP is the pentad of fever, thrombocytopenia, microangiopathic hemolytic anemia, renal insufficiency, and neurologic dysfunction. Educational objective: Heparin-induced thrombocytopenia and thrombosis results from the production of IgG antibodies against complexes of heparin and platelet factor 4. The Fc component of these antibodies binds to platelets, resulting in widespread platelet activation and a prothrombotic state.
A 22-year-old woman comes to the office due to worsening dyspnea and heart pounding with exercise for the last week. She has no chronic medical conditions but reports aches and pains over the past several weeks. The patient takes ibuprofen as needed but no other medications. Temperature is 37.2 C (99 F), blood pressure is 138/86 mm Hg, and pulse is 90/min. BMI is 18 kg/m2. Physical examination shows an erythematous rash in sun-exposed regions. The lungs are clear to auscultation. A midsystolic click and systolic murmur are heard best at the apex without radiation. There is mild tenderness of joints. Laboratory results are as follows: Hemoglobin 7.8 g/dL Reticulocytes 6% Platelets 205,000/mm3 Leukocytes 11,200/mm3 Creatinine 1.4 mg/dL Which of the following is the most likely cause of this patient's hematologic findings? Bone marrow suppression Extravascular hemolysis Intrinsic factor antibodies Thrombotic microangiopathy Traumatic intravascular hemolysis
This patient with dyspnea and heart pounding with exercise has symptomatic anemia. Her other findings, including the presence of joint pain, a photosensitive rash, and mild renal insufficiency, raise suspicion for systemic lupus erythematosus (SLE), an autoimmune disease primarily seen in young women. SLE frequently causes anemia due to a combination of factors, including chronic inflammation (anemia of chronic disease), gastrointestinal serositis (iron deficiency from bleeding), and/or autoimmune hemolytic anemia (AIHA). Approximately 10% of patients with SLE develop AIHA due to immune dysregulation, which results in the formation of IgG autoantibodies against the erythrocyte membrane. Erythrocytes coated with IgG are subsequently identified by the Fc-receptor on splenic macrophages and partially or wholly phagocytized, leading to extravascular hemolysis. Laboratory assessment typically reveals elevated reticulocyte count (ie, reticulocytosis) because interstitial fibroblasts in the kidney sense tissue hypoxia and increase the release of erythropoietin; this drives the bone marrow to increase erythrocytosis, leading to the presence of immature red cells in the peripheral blood (normoblasts, reticulocytes). (Choices A and C) The presence of significant reticulocytosis (eg, >4-5%) rules out causes of anemia associated with an impaired bone marrow response, including bone marrow suppression from cancer or infection (eg, parvovirus), anemia of chronic disease (cytokine-mediated retention of iron), and vitamin deficiency (eg, vitamin B12 deficiency from intrinsic factor antibodies, iron deficiency). (Choice D) A normal platelet count effectively rules out anemia due to a microangiopathic process associated with intravascular platelet consumption (eg, thrombotic microangiopathy). (Choice E) Although traumatic intravascular hemolysis can occur with a mechanical heart valve or severe aortic stenosis, it is uncommon in otherwise healthy young individuals. Furthermore, this patient's heart murmur is most consistent with mitral valve prolapse, which is common in patients with SLE but rarely causes traumatic hemolysis. Educational objective:Anemia with an elevated reticulocyte count (ie, reticulocytosis) indicates that the bone marrow is responding appropriately to the anemia by generating new erythrocytes. Reticulocytosis is commonly seen in patients with hemolysis or acute bleeding. Many other causes of anemia are associated with low reticulocyte count, including bone marrow suppression (eg, parvovirus), iron deficiency anemia, vitamin B12/folate deficiency, and anemia of chronic disease.
A 65-year-old man comes to the office due to a few weeks of progressive exertional dyspnea and fatigue. He has had no chest pain, fever, or cough. The patient has a history of mild aortic stenosis, gastroesophageal reflux disease, obesity, hypertension, and hypercholesterolemia. Temperature is 36.7 C (98 F), blood pressure is 130/78 mm Hg, pulse is 82/min, and respirations are 16/min. BMI is 28 kg/m2. Pulse oximetry shows 96% on room air. Mucosal pallor is present, but there is no jaundice, lymphadenopathy, or jugular venous distension. Cardiopulmonary examination reveals a 2/6 systolic ejection murmur at the second right intercostal area. The abdomen is soft and nontender; there is no palpable hepatosplenomegaly. Peripheral pulses are normal. Laboratory results are as follows: Complete blood count Hemoglobin 7.8 g/dL Platelets 98,000/mm3 Leukocytes 3,800/mm3 Peripheral blood smear is shown below: Hypersegmented neutrophil present Which of the following is most likely responsible for this patient's current condition? Impaired DNA synthesis Impaired heme production Inadequate erythropoietin production Replacement of bone marrow with fat cells Replacement of bone marrow with fibrosis Traumatic hemolysis
This patient with exertional dyspnea, fatigue, conjunctival pallor, and a systolic ejection murmur has symptomatic anemia. The peripheral blood smear findings of macrocytosis (abnormally large red blood cells) and a hypersegmented neutrophil indicate likely megaloblastic anemia. This condition sometimes arises in older individuals who take antacids for gastroesophageal reflux disease, as gastric acid is required for proper vitamin B12 absorption. Vitamin B12 acts as a cofactor in the recycling of 5-methyl-tetrahydrofolate to tetrahydrofolate (reduced form of folate), which serves as a one-carbon donor in the synthesis of amino acids, purines, and thymidine monophosphate. Therefore, folate or vitamin B12deficiency reduces available purines and thymidine, which impairs DNA synthesis and causes cell cycle arrest (in S-phase) or apoptotic cell death. Hematopoietic cells are primarily affected due to their high replication rate. In these cells, nuclear division slows relative to cytoplasmic maturation (nuclear-cytoplasmic dyssynchrony), which results in megaloblastic changes (eg, macrocytosis, hypersegmented neutrophils); in addition, increased apoptosis causes anemia and often mild thrombocytopenia/leukopenia. (Choice B) Iron deficiency anemia is associated with impaired production of heme; however, peripheral blood smear would show microcytosis (not macrocytosis), and hypersegmented neutrophils would not be seen. (Choice C) Chronic kidney disease can reduce production of erythropoietin in peritubular fibroblasts of the kidney, which causes normocytic anemia due to reduced erythropoiesis. However, leukopenia and thrombocytopenia are not typically seen, and peripheral blood smear would not show megaloblastic changes. (Choice D) Aplastic anemia is associated with destruction of hematopoietic stem cells and replacement of the bone marrow with fat cells. Although patients usually have pancytopenia, peripheral blood smear would not show hypersegmented neutrophils. (Choice E) Primary myelofibrosis is a myeloproliferative neoplasm that results in fibrosis of the bone marrow. Anemia is common but patients usually have dacrocytes (tear-drop red blood cells) and nucleated red blood cells on peripheral blood smear, not macrocytosis and hypersegmented neutrophils. (Choice F) Aortic stenosis can cause traumatic hemolysis due to red blood cell shearing. However, peripheral smear generally shows schistocytes, and thrombocytopenia/leukopenia would be unusual. Educational objective: Vitamin B12 is an essential cofactor in the recycling of 5-methyl-tetrahydrofolate to tetrahydrofolate, which is required for amino acid, purine, and thymidine synthesis. Deficiency primarily affects hematopoiesis and is marked by impaired nuclear maturation, leading to macrocytic red blood cells and hypersegmented neutrophils; increased apoptosis often also leads to anemia and mild thrombocytopenia/leukopenia.
A 66-year-old man comes to the office due to several weeks of back and lower extremity pain. The pain feels dull and achy and is worsened by movement, causing him to have difficulty bearing weight and ambulating. He has had no bladder or bowel dysfunction, extremity weakness, or numbness. The patient takes metformin for type 2 diabetes mellitus. He drinks 2 alcoholic beverages daily. Physical examination shows no gross abnormality or deformity of the back or extremities. There is no swelling or redness of the legs, and neurological examination is normal. A radiograph of the right lower extremity is shown in the exhibit. Laboratory results are as follows: Hemoglobin 9.6 g/dL Mean corpuscular volume 92/µm3 Creatinine 1.6 mg/dL Calcium 11.0 mg/dL Glucose 220 mg/dL Total protein 8.8 g/dL Albumin 3.8 g/dL Which of the following is the most likely explanation for this patient's bone changes? Decreased urinary calcium excretion Increased 1,25-dihydroxyvitamin D production Increased secretion of parathyroid hormone Local tumor cell-mediated activation of osteoblasts Local tumor cell-mediated activation of osteoclasts
This patient with mild renal insufficiency, normocytic anemia, hypercalcemia, and bone pain has the classic manifestations of multiple myeloma (MM), a plasma cell malignancy associated with excessive production of monoclonal immunoglobulin (paraprotein). Myeloma cells replicate in the bone marrow and release cytokines that stimulate osteoclasts and inhibit osteoblasts, which leads to the generation of purely osteolytic (radiolucent) bone lesions. Bone remodeling is tightly regulated by osteocytes (stromal bone marrow cells that are embedded within the lacuna-canalicular network) in conjunction with signals from the bone marrow and the bone surface. The RANK/RANKL pathway plays a prominent role. RANK is a transmembrane receptor on the surface of osteoclast precursors, which is activated by RANK-ligand (RANKL) produced by osteocytes. Activation of RANK leads to the differentiation of osteoclast precursors into mature osteoclasts. Myeloma cells stimulate osteoclast maturation by producing RANKL and by destroying osteoprotegerin (OPG), mediated by a secreted proteoglycan. They also uniquely inhibit osteoblast activity by secreting cytokines that block osteoblast function (eg, IL-3, IL-7, Wnt pathway inhibitors). This tilts the balance of bone remodeling strongly toward osteolytic bone destruction, which results in many of the characteristic findings seen in MM, including bone pain, hypercalcemia (liberation of calcium during osteolysis), and purely radiolucent (osteolytic) bone lesions. (Choice A) Decreased urinary calcium excretion can be seen with thiazide diuretic use and can result in hypercalcemia. However, this effect is associated with an increase in bone density; radiolucent bone lesions would not be seen. (Choices B and C) 1,25 dihydroxyvitamin D and parathyroid hormone break down bone by increasing RANKL and decreasing OPG expression. Although vitamin D toxicosis and hyperparathyroidism can be associated with hypercalcemia, they are not associated with anemia or elevations in total serum protein. (Choice D) Several tumors that spread to the bones stimulate osteoclasts, but only myeloma cells also inhibit osteoblasts. Therefore, myeloma cells are one of the few tumors to cause purely osteolytic (radiolucent) lesions. Most other tumors have at least some areas of sclerosis (osteoblastic activity). Educational objective: Multiple myeloma is a plasma cell malignancy associated with purely radiolucent (osteolytic) bone lesions due to the stimulation of osteoclasts and the inhibition of osteoblasts. Myeloma cells stimulate osteoclast development by secreting RANK-ligand and destroying osteoprotegerin, which increases RANK activity and results in osteoclast differentiation.
A 25-year-old woman is brought to the emergency department after a motor vehicle collision. She has a pneumothorax, multiple rib fractures, and a right femur and tibial fracture. A chest tube is placed. The patient receives resuscitation and undergoes repair of the femur and tibial fractures. The following day, she develops respiratory distress, decreased urine output, and persistent bleeding from the chest tube drains and around the intravenous lines. Which of the following sets of laboratory findings is most likely to be seen in this patient? Fibrinogen, Protein C, Factor VII -, -, - -, -, + -, +, + -, +, - +, +, +
This patient with multiple injuries following a severe motor vehicle accident developed respiratory distress, reduced renal output, and oozing from the catheter and venipuncture sites. This presentation raises strong suspicion for disseminated intravascular coagulation (DIC), a consumptive coagulopathy associated with trauma, sepsis, malignancy, and obstetrical complications. In DIC due to trauma, the coagulation cascade is triggered by damage to the vascular endothelium (exposes tissue factor) and tissues (releases procoagulant proteins and phospholipids). This leads to the generation of fibrin- and platelet-rich microvascular thrombi, which consumes platelets, coagulation factors (eg, low factor VII), and fibrinogen. Fibrinolysis and anticoagulant proteins then dissolve the clots, which depletes protein C, protein S, and antithrombin. Most patients with acute DIC develop bleeding complications such as oozing from catheters/venipuncture sites, ecchymosis, and petechiae. End organ damage to the kidneys (eg, renal insufficiency) and lungs (eg, pulmonary hemorrhage) is common. Many patients also have microangiopathic hemolytic anemia (eg, schistocytes) due to shearing of red blood cells by intravascular thrombi. Educational objective: Acute disseminated intravascular coagulation is a consumptive coagulopathy linked to severe trauma. Widespread formation of microvascular thrombi leads to consumption of platelets, coagulation factors, and fibrinogen. Subsequent activation of anticoagulant proteins leads to low protein C/S; fibrinolysis elevates D-dimer. Most patients have bleeding complications (eg, oozing from venipuncture/catheter sites) and end organ damage to the lungs or kidneys.
A 9-year-old boy with a history of sickle cell disease is brought to the office due to fatigue. The patient takes penicillin, hydroxyurea, and folic acid daily. Temperature is 37.1 C (98.8 F), blood pressure is 102/68 mm Hg, pulse is 130/min, and respirations are 20/min. Examination shows conjunctival pallor. The oropharynx is clear and the neck is supple. The lungs are clear to auscultation bilaterally, and the abdomen is soft with normal bowel sounds; there is no organomegaly. Skin examination is unremarkable. Laboratory results are as follows: Hemoglobin 5.8 g/dL Mean corpuscular volume 116 µm3 Reticulocytes 0.5% Platelets 90,000/mm3 Leukocytes 3,000/mm3 Which of the following is the most likely cause of this patient's findings? Antibiotic-associated autoimmune hemolysis Decreased methyl group involved in DNA synthesis Loss of complement inhibition in hematopoietic stem cells Medication-induced inhibition of a nucleic acid synthesis enzyme Sequestration of cells in the reticuloendothelial system
This patient with sickle cell disease (SCD) has pancytopenia, a decrease in all 3 blood cell lineages. Pancytopenia is caused by hematopoietic stem cell dysfunction, bone marrow infiltration, or peripheral destruction of mature blood cells. Important clues in this case include: Low reticulocyte count: Patients with severe anemia usually have increased reticulocytes due to enhanced bone marrow erythropoiesis. The presence of pancytopenia with an inappropriately low reticulocyte count usually indicates the bone marrow is impaired and is unable to adequately generate new cells despite strong growth signals (eg, increased erythropoietin). Macrocytosis: Although macrocytosis can be seen in a wide range of pathologies, the presence of pancytopenia plus macrocytosis usually indicates a vitamin deficiency (eg, vitamin B12, folate) or a medication (eg, hydroxyurea) is impairing DNA synthesis in hematopoietic cells. Hydroxyurea inhibits ribonucleoside reductase, an enzyme that generates deoxyribonucleoside triphosphates for DNA synthesis/repair. A lack of deoxyribonucleoside triphosphates slows erythrocyte nuclear development, leading to decreased cellular division with an overall increase in erythrocyte size. High doses of hydroxyurea also impair nuclear development in precursors of leukocytes and thrombocytes, leading to pancytopenia. Hydroxyurea is the major disease-modifying medication in SCD; it reduces production of the mutated beta globulin gene and increases production of the fetal hemoglobin gene, which increases fetal hemoglobin (HbF). Because HbF functions normally, erythrocytes with increased HbF are less likely to sickle in the microvasculature. (Choice A) Autoimmune hemolytic anemia is associated with high (not low) reticulocytes due to increased erythropoiesis. Because reticulocytes are larger (MCV 125-135 µm3) than mature erythrocytes, reticulocytosis can cause macrocytosis; however, leukopenia and thrombocytopenia would not be seen. (Choice B) Folate is a methyl group donor for the synthesis of purines and pyrimidines. Folate deficiency impairs DNA synthesis, which can lead to pancytopenia and macrocytosis. However, this patient is taking folic acid, making deficiency unlikely. (Choice C) Paroxysmal nocturnal hemoglobinuria is an acquired disorder of hematopoietic stem/progenitor cells marked by lack of complement inhibitor on the erythrocyte surface. Hemolytic anemia is the primary manifestation; reticulocytosis is common, and pancytopenia rarely occurs. (Choice E) SCD can cause hypersplenism due to splenic congestion with damaged erythrocytes. Although this can occasionally cause pancytopenia, patients with hypersplenism usually have splenomegaly and normocytic (not macrocytic) anemia. Educational objective: Hydroxyurea increases fetal hemoglobin production, which reduces erythrocyte sickling in patients with sickle cell disease. It also inhibits ribonucleoside reductase, which decreases deoxynucleoside triphosphates available for DNA synthesis; this leads to macrocytosis and, with high doses, pancytopenia.
A 35-year-old woman comes to the office for evaluation of fatigue and exertional dyspnea. She has a history of systemic lupus erythematosus and is noncompliant with therapy. The patient takes naproxen as needed for joint pains. Vital signs are within normal limits. Examination shows mild pallor. The remainder of the examination shows no abnormalities. Laboratory results are as follows: Hemoglobin 8.6 g/dL Mean corpuscular volume 80 mm3 Creatinine 0.8 mg/dL Iron, serum 40 µg/dL Total iron-binding capacity 180 µg/dL(normal: 250-460 µg/dL) Lactate dehydrogenase 72 U/L Which of the following is the most likely underlying cause of her current condition? Immune hemolysis of erythrocytes Increased cellular use of folic acid Intrinsic factor antibodies Iron sequestration of macrophages Occult gastrointestinal blood loss
This patient with untreated systemic lupus erythematosus (SLE) has normocytic anemia with low serum iron and low total iron-binding capacity, raising strong suspicion for anemia of chronic disease (ACD). Most cases arise in those with chronic elevation of inflammatory cytokines due to underlying rheumatologic disease (eg, SLE), chronic infection, or malignancy. Although a number of inflammatory cytokines contribute to the development of ACD, the primary mediator is hepcidin, a small peptide released by the liver in response to inflammation. Hepcidin binds to and inactivates iron channels (ferroportin) on enterocytes and reticuloendothelial macrophages, which results in reduced iron absorption in the gut and increased iron sequestration in the reticuloendothelial system. Because reticuloendothelial macrophages recycle senescent erythrocytes and provide >95% of daily iron for erythrocytosis, sequestration of iron in the reticuloendothelial system dramatically reduces serum iron concentration. This limits the amount of iron available for the generation of new erythrocytes and typically results in a normocytic (or slightly microcytic) anemia with a low reticulocyte response. In ACD, total iron-binding capacity is generally normal or slightly reduced due to cytokine-mediated suppression of transferrin. (Choices A and E) Patients with SLE can develop immune-mediated hemolysis due to the generation of autoantibodies against the erythrocyte membrane; however, this typically results in extravascular hemolysis, which increases the intracellular enzyme lactate dehydrogenase. The presence of normal lactate dehydrogenase in this case makes hemolysis less likely. Patients with SLE also sometimes develop gastrointestinal bleeding from serositis (or chronic NSAID use), which can lead to iron deficiency anemia. However, iron deficiency anemia is typically associated with microcytic anemia and a high total iron-binding capacity (because transferrin levels are markedly increased). (Choices B and C) Conditions associated with rapid cell turnover can result in folate deficiency due to increased folate use during the generation of new cells. Intrinsic factor antibodies are a common cause of vitamin B12 deficiency. However, folate and vitamin B12 deficiency are generally marked by macrocytic anemia with a normal or high serum iron level. Educational objective: Anemia of chronic disease is a consequence of chronically elevated inflammatory cytokines, most importantly hepcidin. This peptide inactivates iron channels on enterocytes and reticuloendothelial macrophages, leading to reduced iron absorption and reduced iron release from the reticuloendothelial system. The net result is normocytic or slightly microcytic anemia with low reticulocyte response, low serum iron level, and normal or low total iron-binding capacity.
A 68-year-old man comes to the emergency department due to lightheadedness, generalized weakness, and palpitations. The patient has a prolonged history of hypertension and takes amlodipine. He does not use tobacco or alcohol. His blood pressure is 110/60 mm Hg and pulse is 144/min and irregular. ECG shows an irregularly irregular rhythm and absent P waves. The patient is started on beta blocker therapy for rate control, with improvement in his heart rate. Long-term anticoagulation is initiated to prevent atrial thrombus formation, and he is eventually discharged home on warfarin. Which of the following is the best test to monitor the anticoagulation effect of warfarin in this patient? Activated partial thromboplastin time Bleeding time Fibrin split products Fibrinogen levels Prothrombin levels
This patient's ECG (irregularly irregular rhythm with absent P waves) suggests atrial fibrillation (AF), which alters cardiac flow dynamics, making thrombus formation more common. If this thrombus detaches from the atrial wall, there is a risk of stroke. The oral anticoagulant warfarin (Coumadin) reduces thrombus formation risk by inhibiting the activation of vitamin K-dependent clotting factors II, VII, IX, and X (vitamin K antagonist). This leads to decreased levels of these factors, particularly factor VII, and therefore prolongs the prothrombin time (PT). Another useful laboratory value is the International Normalized Ratio (INR), a ratio of the patient's PT to a control (usual INR target for AF: 2.0-3.0). Examples of indications include AF, deep venous thrombosis, and pulmonary thromboembolism. Outpatients who take warfarin typically have their INR checked every few weeks (more frequently initially). The most common side effect is excessive anticoagulation leading to bleeding (which can require vitamin K and fresh frozen plasma if rapid reversal is needed). Skin necrosis can occur during the first few days of treatment due to a transient hypercoagulable state (reduction in protein C anticoagulant activity before affecting other factors' procoagulant activity); for this reason, heparin is typically administered temporarily at the start of warfarin therapy ("heparin bridge"). Warfarin should not be used during pregnancy because it is teratogenic and can cause fetal bleeding. (Choice A) Activated partial thromboplastin time (aPTT) is used to monitor unfractionated heparin, which primarily affects the intrinsic coagulation pathway. (Choice B) Bleeding time is used for the assessment of platelet function. Increased bleeding time can be seen with thrombocytopenia, von Willebrand disease, defects of platelet aggregation, disseminated intravascular coagulation (DIC), and aspirin therapy. (Choices C and D) The combination of a decreased fibrinogen level and the presence of fibrin split products (such as D-dimer) is characteristic of DIC. Educational objective:Warfarin is an oral anticoagulant that inhibits the carboxylation of vitamin K-dependent coagulation factors II, VII, IX, and X. It is used in atrial fibrillation, deep venous thrombosis, and pulmonary thromboembolism. Prothrombin time (PT)/International Normalized Ratio (INR) should be monitored regularly during treatment with warfarin. Activated partial thromboplastin time (aPTT) is used for monitoring unfractionated heparin. Educational objective: Warfarin is an oral anticoagulant that inhibits the carboxylation of vitamin K-dependent coagulation factors II, VII, IX, and X. It is used in atrial fibrillation, deep venous thrombosis, and pulmonary thromboembolism. Prothrombin time (PT)/International Normalized Ratio (INR) should be monitored regularly during treatment with warfarin. Activated partial thromboplastin time (aPTT) is used for monitoring unfractionated heparin.
A 3-year-old boy is brought to the physician with jaundice and pallor. He was adopted and his family history is unknown. Laboratory studies show anemia, reticulocytosis, and increased indirect bilirubin. A peripheral blood smear shows red blood cells without central pallor. This patient is most likely to have which of the following additional findings? Decreased lactate dehydrogenase Increased haptoglobin Increased mean corpuscular hemoglobin concentration Increased mean corpuscular volume Red blood cell inclusions
This patient's anemia, reticulocytosis, and increased indirect bilirubin are indicative of hemolytic anemia. His peripheral blood smear shows red blood cells without central pallor (ie, spherocytes), which suggests acquired or hereditary spherocytosis. Hereditary spherocytosis is caused by defective binding of the red cell cytoskeleton to the plasma membrane due to mutations involving ankyrin, band 3, or spectrin proteins; acquired spherocytosis is most often caused by autoimmune hemolytic anemia. Compared to normal red blood cells, spherocytes are smaller and have a more intensely-staining cytoplasm due to membrane loss and red cell dehydration. The red cell index most specific for spherocytosis is an elevation in mean corpuscular hemoglobin concentration (MCHC). (Choice A) Lactate dehydrogenase (LDH) is an enzyme present in red blood cells. LDH will be elevated in any cause of hemolytic anemia. (Choice B) Haptoglobin is an acute phase reactant that combines with free hemoglobin in the circulation to preserve body iron stores and prevent tissue damage. Haptoglobin levels decrease in any form of hemolytic anemia (even with extravascular hemolysis, as some free hemoglobin will spill into the blood stream). (Choice D) The mean corpuscular volume (MCV) is generally normal to low in hereditary spherocytosis. (Choice E) Red blood cell inclusions, such as Howell-Jolly bodies (DNA) and Pappenheimer bodies (iron), are occasionally associated with hemolytic anemia but are more commonly associated with splenectomy and sideroblastic anemia, respectively. Educational objective: In spherocytosis, mean corpuscular hemoglobin concentration (MCHC) is increased due to mild dehydration of the red blood cell. Markers of hemolysis are often evident and include elevated lactate dehydrogenase, reticulocytosis, and decreased haptoglobin.
A 68-year-old woman comes to the office due to recurrent bacterial sinusitis, pneumonia, and urinary tract infections over the past several months. The patient has no prior medical conditions or history of severe infections in childhood. She does not use tobacco, alcohol, or illicit drugs. Temperature is 36.6 C (97.9 F), blood pressure is 130/76 mm Hg, and pulse is 72/min. On physical examination, the lungs are clear to auscultation, heart sounds are normal, and the abdomen is soft and nontender with no hepatosplenomegaly. Laboratory testing reveals normocytic normochromic anemia. Bone marrow aspirate shows an increased number of cells, as shown below (clock-faced plasma cells). Which of the following is the most likely cause of this patient's increased susceptibility to infections? Aberrant lysosomal transport and fusion in leukocytes Decreased production of functional immunoglobulins Defective adhesion and transmigration of leukocytes Impaired formation of membrane attack complex Impaired phagocytic ability to generate superoxide
This patient's bone marrow aspirate has numerous plasma cells, which are distinguished by the presence of abundant basophilic cytoplasm, perinuclear paleness (large Golgi apparatus), and nuclei with "clock-face" (peripheral) chromatin. The presence of >10% plasma cells in the bone marrow raises strong suspicion for multiple myeloma (MM). In MM, neoplastic plasma cells crowd the bone marrow, leading to impairments in normal hematopoiesis. B-cell development is particularly affected, which reduces plasma cell diversity and limits the generation of targeted immunoglobulins against infectious agents. Therefore, patients with MM have increased risk of recurrent bacterial infections, particularly of the urinary tract, lungs, and sinuses. Although patients with MM have high levels of circulating monoclonal immunoglobulin, monoclonal immunoglobulin does not provide immunity against a diverse range of pathogens. MM is also associated with normocytic anemia (due to ineffective erythropoiesis), bone pain (due to osteolytic lesions), hypercalcemia, and renal insufficiency (due to clogging of the tubules with immunoglobulin light chains). (Choice A) Chediak-Higashi syndrome is a rare autosomal recessive disorder that causes abnormal lysosomal fusion and transport. Although it is associated with recurrent bacterial infections and hematologic malignancies, most cases are identified in early childhood, and patients generally have hypopigmentation and progressive neurologic dysfunction. (Choice C) Leukocyte adhesion deficiency is an inherited disorder associated with defective adhesion and transmigration of leukocytes from the vasculature to tissues. Patients typically have recurrent bacterial infections that begin in early childhood. It is not associated with excessive plasma cells in the bone marrow. (Choice D) Terminal complement deficiency (C5-C9) prevents the formation of the membrane attack complex and dramatically increases the risk of recurrent infection with Neisseria species; patients are usually healthy otherwise and have no abnormalities in the bone marrow. (Choice E) Chronic granulomatous disease is a genetic disorder associated with severe bacterial and fungal infections due to impaired ability of phagocytes to generate reactive oxygen species (oxidative burst). Most cases present in early childhood, and plasma cell effacement of the bone marrow would not be seen. Educational objective: Multiple myeloma is associated with the clonal proliferation of plasma cells in the bone marrow. This prevents normal B-cell lymphogenesis and increases the risk of bacterial infection due to decreased production of normal immunoglobulins.
A 19-year-old woman comes to the office due to exertional dyspnea and intermittent lightheadedness. On examination, mucosal pallor is present. Laboratory studies reveal a red blood cell count of 3.5 million/mm3 and a hemoglobin concentration of 8.5 g/dL. Peripheral smear is shown in the image below: ID=1935 review Which of the following findings are most likely present in this patient? Decreased transferrin binding capacity Increased macrophage iron retention Increased serum cytokine levels Increased serum erythropoietin levels Increased serum hepcidin levels
This patient's dyspnea, lightheadedness, and mucosal pallor are likely due to symptomatic anemia. A number of underlying processes can cause anemia, but the differential diagnosis can usually be narrowed by concurrent clinical data. In this case, the peripheral blood smear demonstrates hypochromic, microcytic erythrocytes and a variety of erythrocyte shapes (poikilocytosis), raising strong suspicion for severe iron deficiency anemia (IDA). Young women often develop IDA from heavy menstruation. Patients with severe IDA do not have enough iron to generate adequate hemoglobin for each erythrocyte, which results in low mean corpuscular hemoglobin concentration (hypochromia) and small erythrocyte size (microcytosis). Interstitial fibroblasts in the peritubular capillaries and proximal convoluted tubule of the kidney detect the reduced oxygen carrying capacity of the blood (mild tissue hypoxia) and increase erythropoietin release to drive the generation of new erythrocytes. However, new erythrocytes cannot be produced at a level to compensate for the anemia due to insufficient serum iron; therefore, patients typically have minimal reticulocytes and low red blood cell counts in the peripheral blood. (Choice A) Transferrin is a serum protein that binds free iron. The concentration of transferrin is increased in IDA (to maximize free iron scavenge), but the amount of iron bound to it is reduced, leading to a low transferrin saturation on laboratory evaluation. (Choices B, C, and E) Hepcidin is a small peptide released by the liver in response to inflammation. It binds to and inhibits iron channels on the surface of enterocytes and on reticuloendothelial macrophages, which reduces iron absorption in the gut and reduces iron release from the reticuloendothelial system (the primary site of iron recycling). Because IDA is not mediated by inflammation, hepcidin levels are typically low; this increases gastrointestinal iron absorption and iron release from the reticuloendothelial system (eg, splenic macrophages). In contrast, anemia of chronic disease is associated with very high levels of hepcidin (and other inflammatory cytokines), which reduces iron availability for erythrocytosis despite adequate total body stores of iron. Patients with anemia of chronic disease typically have normocytic anemia and normochromia (not hypochromia). Educational objective: Patients with severe iron deficiency anemia (IDA) typically have evidence of hypochromic, microcytic erythrocytes on peripheral blood smear with poikilocytosis. Laboratory studies will reveal low red blood cell count, low hepcidin, and high transferrin. Although erythropoietin is markedly elevated in IDA, there is not enough iron to efficiently generate erythrocytes; therefore, reticulocyte count is low.
A 12-year-old boy experiences excessive bleeding following a tooth extraction. His past medical history includes episodes of painful joint swelling from minor trauma. Physical examination shows a soft and nontender abdomen with a liver span of 10 cm. The spleen is not palpable. Laboratory results are as follows: Bleeding time 5 min Activated partial thromboplastin time 25 sec Prothrombin time 23 sec This patient most likely has a deficiency involving which of the following factors? Factor VII Factor VIII Factor XI Hageman factor von Willebrand factor
This patient's hemarthroses and excessive bleeding following tooth extraction are suggestive of a coagulopathy (ie, clotting factor deficiency). Coagulopathies generally present with deep-tissue bleeding into joints, muscles, and subcutaneous tissue. In contrast, platelet defects typically manifest with mucocutaneous bleeding (eg, epistaxis, petechiae). The patient's laboratory results show a normal bleeding time, normal activated partial thromboplastin time (aPTT), and a prolonged prothrombin time (PT). The normal bleeding time indicates adequate platelet function, and the normal aPTT indicates an intact intrinsic coagulation system. However, the prolonged PT indicates a defect in the extrinsic coagulation pathway. Therefore, the patient is most likely deficient in factor VII. (Choice B) Factor VIII deficiency results in classic X-linked hemophilia A. Coagulation studies will show a normal bleeding time, normal PT, and prolonged aPTT. (Choice C) Factor XI deficiency results in a rare autosomal recessive disorder called hemophilia C. Spontaneous bleeding is rare in this condition, but affected patients are at increased risk for bleeding following surgery or trauma. Coagulation studies will show a prolonged aPTT and normal PT. (Choice D) Hageman factor (factor XII) deficiency is an autosomal recessive disorder that does not typically cause clinically significant bleeding despite aPTT prolongation. (Choice E) von Willebrand factor (vWF) functions as both a carrier molecule for factor VIII (protecting it from being degraded) and as a mediator of platelet adhesion to the endothelium. It presents with prolonged bleeding time and normal/prolonged aPTT. Educational objective: A normal bleeding time indicates adequate platelet hemostatic function. A normal activated partial thromboplastin time (aPTT) indicates an intact intrinsic coagulation system. Prolonged prothrombin time in the setting of normal aPTT indicates a defect in the extrinsic coagulation system at a step that is not shared with the intrinsic system.
A 45-year-old man comes to the office due to 3 months of exertional fatigue and weight loss. He is sexually active with multiple male partners. Temperature is 37.9 C (100.2 F), blood pressure is 110/70 mm Hg, pulse is 84/min, and respirations are 13/min. BMI is 20 kg/m2. The patient appears chronically ill and has mucosal pallor. There are thick, white plaques over the buccal mucosa and soft palate. Anterior and posterior cervical lymphadenopathy is present. The lungs are clear to auscultation. The abdomen is soft and nontender. There is no organomegaly. Laboratory results are as follows: Hematocrit 28% Mean corpuscular volume 76 µm3 Platelets 160,000/mm3 Leukocytes 4,100/mm3 Iron, serum 30 µg/dL Total iron-binding capacity 190 µg/dL (normal: 250-460) Which of the following is the most likely underlying cause of this patient's anemia? Abnormal iron utilization Decreased synthesis of globin chains Deposition of fibrin in the microcirculation Gastrointestinal blood loss Deficiency of a heme synthesis enzyme
This patient's high-risk sexual history, weight loss, lymphadenopathy, chronically ill appearance, and oral candidiasis (white buccal/soft palate plaques) suggest undiagnosed HIV infection. Patients with chronic infections such as HIV often develop anemia of chronic disease (ACD) due to high circulating levels of inflammatory cytokines, which cause maladaptive changes to iron utilization. Although ACD is mediated by a wide range of inflammatory cytokines (eg, IL-1, IL-6, tumor necrosis factor-alpha, interferon gamma), the major player is hepcidin, a small peptide released by the liver in response to inflammation. Hepcidin binds to and inhibits iron channels on enterocytes and reticuloendothelial macrophages, leading to reduced iron absorption in the gut and reduced iron recycling in the reticuloendothelial system (the site of senescent erythrocyte destruction). This significantly reduces circulating serum iron, which limits iron availability for erythropoiesis. Patients with ACD typically develop normocytic or slightly microcytic anemia with a low reticulocyte response. Total iron-binding capacity is generally reduced due to cytokine-mediated suppression of transferrin. Treatment of the underlying condition (eg, antiretroviral therapy) often ameliorates or resolves ACD. (Choice B) Decreased synthesis of globin chains is seen in thalassemia, a genetic disorder that presents with microcytic anemia and reduced erythrocyte life span. Serum iron levels tend to be high with thalassemia due to increased erythropoiesis to compensate for the anemia. (Choice C) Fibrin deposition in the microvasculature is seen in disseminated intravascular coagulation, a consumptive coagulopathy marked by microangiopathic hemolytic anemia and significant thrombocytopenia. Patients typically have normocytic (not microcytic) anemia and normal or high serum iron levels. (Choice D) Iron deficiency anemia (eg, gastrointestinal bleeding) also typically causes microcytic anemia with low serum iron. However, unlike ACD, iron deficiency anemia is typically associated with a high iron-binding capacity due to increased circulating transferrin levels. (Choice E) Porphyria is a genetic disorder caused by deficiency of heme synthesis enzymes. Patients typically develop neurovisceral (eg, abdominal pain) or cutaneous (eg, blistering) manifestations. Microcytic anemia with low serum iron would be atypical. Educational objective: Anemia of chronic disease is caused by chronic elevations in inflammatory cytokines, which lead to reduced iron absorption and increased iron retention in the reticuloendothelial system. This impairs iron availability and causes a normocytic or slightly microcytic anemia with low serum iron and low total iron-binding capacity.
A 5-year-old boy is brought to the emergency department due to an hour of epistaxis after he was elbowed in the face during a basketball game. He has a history of prolonged bleeding from the gums after brushing his teeth. Vital signs are normal. The patient has scattered bruises along his arms and legs. He receives desmopressin, which stops the bleeding. This treatment most likely improved the patient's condition through which of the following therapeutic mechanisms? Binds to antidiuretic hormone receptors Increases circulating factor IX Increases circulating von Willebrand factor Increases platelet count Increases vitamin K-dependent coagulation factor Relaxes vascular smooth muscle cells
This patient's history of prolonged mucosal bleeding (eg, oropharyngeal, nasal), easy bruising, and responsiveness to desmopressin (DDAVP) likely indicate von Willebrand disease (vWD), the most common inherited bleeding disorder. VWD is caused by qualitative or quantitative defect in von Willebrand factor (vWF), a multimeric glycoprotein synthesized by endothelial cells and platelets. VWF contributes to hemostasis by forming a bridge between platelets and exposed subendothelial components at sites of injury and by serving as a carrier protein for factor VIII (whose half-life is tremendously shortened when not bound to vWF). Treatment of minor bleeding episodes in patients with vWD often involves desmopressin (DDAVP), a synthetic analogue of antidiuretic hormone that does not induce vasoconstriction. DDAVP induces a rapid and transient increase in vWF and factor VIII by indirectly releasing these compounds from endothelial storage sites. DDAVP is most useful in the acute setting, as repeated doses have limited effectiveness due to tachyphylaxis (endothelial stores become depleted). (Choices A and F) Antidiuretic hormone binds to the vasopressin 2 receptor on renal tubular cells, which leads to increased aquaporin channels, increased water reabsorption, and decreased urine output. It also binds to the vasopressin 1 (V1) receptor on vascular smooth muscle, which results in vasoconstriction. In contrast, DDAVP has antidiuretic effects but little or no vasoactive (vasoconstrictive) effects because it only binds to the V2 receptor. (Choice B) Factor IX, a vitamin K-dependent clotting factor produced in the liver, is deficient in hemophilia B. DDAVP has no effect on factor IX levels. Hemophilia B must be treated with factor IX replacement. (Choice D) Platelets form the initial hemostatic plug when bleeding occurs. DDAVP has no effect on platelet number. (Choice E) Patients with vitamin K deficiency are prone to bleeding due to decreased vitamin K-dependent coagulation factors (factors II, VII, IX, and X). Vitamin K deficiency is typically seen in patients with fat malabsorption (eg, pancreatic insufficiency) or those on warfarin, a vitamin K antagonist. Educational objective: Patients with symptomatic von Willebrand disease can be treated with desmopressin (DDAVP), a synthetic antidiuretic hormone analogue that has no vasoconstrictive effects. DDAVP transiently increases von Willebrand factor and factor VIII release from endothelial cell storage through indirect mechanisms; it can be used prophylactically prior to procedures or to treat minor bleeding.
A 78-year-old man is found unresponsive and is brought by ambulance to the emergency department. Upon arrival, the family says he has a complicated medical history and takes multiple medications but has no known drug allergies. Noncontrast CT scan of the head reveals intracranial hemorrhage. The patient is given a recombinant biologic agent that has antigen homology with factor Xa but no catalytic effect. The agent is most likely to antagonize the effects of which of the following drugs? Apixaban Aspirin Dabigatran Ticagrelor Warfarin
This patient's intracranial hemorrhage was likely treated with andexanet alfa, a biologic agent that shares homology with factor Xa but has no proteolytic effect. It is administered to patients who have life-threatening bleeding while on a factor Xa inhibitor (eg, rivaroxaban, apixaban). The similarity of andexanet to factor Xa allows it to function as a decoy that binds to factor Xa inhibitors. This restores intravascular coagulation by increasing the availability of endogenous factor Xa, which converts prothrombin to thrombin and generates fibrin clots. Andexanet is reserved for life-threatening bleeding because it is associated with a significant risk of thrombosis. (Choice B) Aspirin inhibits cyclooxygenase, which prevents platelet aggregation by blocking the generation of thromboxane A2. There is no reversal agent for aspirin widely available. Aspirin does not target factor Xa. (Choice C) Dabigatran is a direct oral anticoagulant (DOAC) that inhibits circulating and clot-bound thrombin. It is reversed by idarucizumab, a monoclonal antibody fragment that binds to and inhibits dabigatran. Although dabigatran is a DOAC, it targets thrombin rather than factor Xa. (Choice D) Ticagrelor blocks the P2Y12 adenosine diphosphate receptor on the surface of platelets, which prevents platelet aggregation. Its activity is not mediated by factor Xa. (Choice E) Warfarin is a vitamin-K antagonist that prevents the generation of vitamin-K dependent clotting factors by blocking epoxide reductase in the liver. It is reversed by the administration of vitamin K. Educational objective: Andexanet alfa is a factor Xa decoy that has no proteolytic effect. It is administered to patients on factor Xa inhibitors (eg, rivaroxaban, apixaban) who have life-threatening bleeding in order to reverse the anticoagulation effect.
A 70-year-old man develops progressive fatigue and easy bruising. He has had unintentional weight loss and episodic fevers. Physical examination shows splenomegaly and enlarged lymph nodes in the cervical, axillary, and inguinal regions. Complete blood count reveals anemia, thrombocytopenia, and leukocytosis. Peripheral blood smear shows an increased number of small, mature-appearing, abnormal lymphocytes, which express CD20 and kappa light chains. During treatment, the patient receives an agent that directly binds and inhibits BCL-2 protein. Which of the following is the most likely effect of this therapy on the abnormal cells? Accumulation of ubiquintinated intracellular proteases Greater susceptibility to cytotoxic T cells Increased activation of caspases Increased single-strand DNA breaks Interruption of cell growth in S phase Targeting of antibody-dependent cellular cytotoxicity
This patient's lymphadenopathy, splenomegaly, and increased number of mature lymphocytes in the blood that express B-cell markers (CD20+, kappa light chains) indicate chronic lymphocytic leukemia (CLL). This disease is characterized by the progressive accumulation of mature B cells in hematopoietic tissues (leading to anemia, thrombocytopenia, lymphadenopathy, and splenomegaly) and peripheral blood (leading to leukocytosis) due to genetic mutations that lengthen B-cell survival. In most cases of CLL, B-cell survival is promoted by the overexpression of BCL-2 on the mitochondrial membrane. BCL-2 is an antiapoptotic protein that prevents stress signals from triggering the intrinsic apoptotic cascade, which is mediated by the release of cytochrome c from the mitochondria and the subsequent activation of caspases. Treatment of CLL with BCL-2 inhibitors (eg, venetoclax) makes tumor cells more sensitive to stress signals (eg, chemotherapy) by increasing the activation of caspases, which leads to cell death. (Choice A) Bortezomib, a proteasome inhibitor that prevents the destruction of ubiquinated intracellular proteins, can be used against neoplasms that generate excessive amounts of protein (eg, multiple myeloma). (Choice B) Because cancer cells produce new antigens (neoantigens) not seen in normal cells, they generally have mutations that allow them to evade cytotoxic T-cell detection. Immune checkpoint inhibitors, such as antibodies against PD-1 (eg, pembrolizumab) or CTLA-4, prevent cancer cells from evading cytotoxic T-cell detection. (Choice D) Olaparib inhibits poly-ADP ribose polymerase, the enzyme that mediates the repair of single-strand DNA breaks. It is often used to treat breast, ovarian, or prostate cancer in individuals with BRCA mutations. (Choice E) Many forms of chemotherapy target rapidly dividing cells by inhibiting DNA replication during S phase (eg, purine analogs). BCL-2 does not mediate DNA replication or repair. (Choice F) Rituximab is a monoclonal antibody against the B-cell surface marker CD20. Although it is often used for CLL, it mediates cell death by binding to the B-cell surface (antibody-mediated cytotoxicity). Educational objective: Chronic lymphocytic leukemia is associated with overexpression of BCL-2, an anti-apoptotic protein, on the mitochondrial membrane. Treatment with a BCL inhibitor causes cancer cell death by promoting cytochrome c release from the mitochondria, which subsequently activates caspases.
A 67-year-old man is evaluated for persistent lower back pain and fatigue. The patient has no other medical conditions and takes no medications. Laboratory testing shows anemia and renal dysfunction. Serum total protein levels are elevated, and electrophoresis of the serum proteins reveals a narrow spike in the gamma globulin region. A subsequent bone marrow biopsy shows abnormal marrow cells. A medication that blocks cellular proteasome action is administered. This treatment is most likely to cause which of the following effects on the abnormal cells? Augmented cytotoxic T-cell response Decreased DNA methylation Impaired RNA splicing Improved differentiation Increased apoptosis
This patient's manifestations (anemia, back pain, renal insufficiency), monoclonal protein spike on electrophoresis, and bone marrow findings indicate likely multiple myeloma (MM), a plasma cell malignancy associated with the overproduction of monoclonal immunoglobulins or IgG fragments (eg, IgG light chains). Treatment regimens for MM often incorporate medications that block proteasome function (eg, bortezomib). The proteasome is a barrel-shaped structure with a proteolytic core that degrades ubiquitin-tagged proteins. Proteasome inhibition results in the accumulation of damaged and misfolded proteins that have a cytotoxic effect. MM cells are particularly susceptible to proteasome inhibition because they generate extremely high quantities of secretory proteins (eg, monoclonal immunoglobulins), some of which misfold and aggregate within the endoplasmic reticulum. Increased stress on the endoplasmic reticulum triggers an unfolded protein response that eventually leads to activation of caspases and cellular apoptosis. (Choice A) Cytotoxic T-cell destruction of cancer cells is augmented by checkpoint inhibitor immunotherapies such as those that target the programmed cell death 1 receptor and CTLA-4. These drugs are often used in the treatment of MM but augmented cytotoxic T-cell function is not related to proteosome inhibition. (Choice B) Azacitidine is a cytidine analogue that inhibits DNA methyltransferase. It reduces DNA methylation and alters gene expression, leading to reduced protein synthesis and increased cellular differentiation. It is toxic to abnormal hematopoietic cells but is used primarily to treat myelodysplastic syndrome. (Choice C) RNA splicing is done by the nuclear spliceosome, which removes introns from transcribed pre-mRNA. The proteasome does not affect RNA splicing. (Choice D) All-trans-retinoic acid is used in the treatment of acute promyelocytic leukemia. It works by promoting differentiation of neoplastic cells into mature neutrophils. Educational objective: Multiple myeloma, a plasma cell malignancy associated with significant production of secretory proteins (eg, monoclonal immunoglobulins), is frequently treated with proteasome inhibitors (eg, bortezomib). These drugs block the degradation of ubiquitinated proteins by the proteosome, which leads to accumulation of abnormally folded proteins that trigger cellular apoptosis.
A 54-year-old woman comes to the office due to 3 months of progressive pain and prolonged morning stiffness in her hand joints. She has tried over-the-counter naproxen without significant relief. Review of systems is positive for fatigue. The patient has no other chronic medical conditions and takes no other medications regularly. Vital signs are normal. Swelling and tenderness of the proximal interphalangeal joints and metacarpophalangeal joints are present in both hands. Laboratory studies reveal a hemoglobin of 9.8 g/dL and MCV of 86. The patient is prescribed methotrexate therapy. Laboratory results 6 months later reveal that hemoglobin has increased to 12 g/dL. If measured, which of the following changes were most likely present in this patient before the start of methotrexate therapy? Circulating iron level, bone marrow iron low, high low, low high, low high, high normal, low high, normal
This patient's polyarticular hand joint pain, swelling, and prolonged morning stiffness are suggestive of rheumatoid arthritis (RA). Patients with chronic rheumatologic diseases such as RA have high circulating levels of inflammatory cytokines, which can trigger anemia of chronic disease (ACD) due to maladaptive changes in iron utilization. ACD is primarily mediated by hepcidin, a small peptide released by the liver in response to inflammation. Hepcidin binds to and inactivates iron channels (ferroportin) on enterocytes and reticuloendothelial macrophages, leading to reduced iron absorption in the gut and reduced iron recycling by the reticuloendothelial system (the site of senescent erythrocyte destruction and the source of >95% of daily iron for new RBC production). These changes significantly reduce circulating iron levels, which impairs the generation of heme for new erythrocytes. Although patients with suspected ACD do not generally require bone marrow biopsy for diagnosis, it would show increased sequestration of iron within macrophages and reduced quantities of erythrocyte precursor cells (eg, normoblasts, reticulocytes). Patients with ACD typically develop a normocytic (or slightly microcytic) anemia with low reticulocyte count. Treatment of the underlying inflammatory condition (eg, methotrexate for RA) reduces serum inflammatory cytokine levels and generally improves the anemia. (Choice B) In contrast to ACD, patients with iron deficiency anemia have reduced serum iron levels and reduced bone marrow iron levels due to low total body iron stores. Educational objective: Anemia of chronic disease is caused by elevated inflammatory cytokine levels (eg, hepcidin), which increase sequestration of iron within the reticuloendothelial system and reduce circulating iron levels. Many patients have improvement in anemia with treatment of the underlying condition that caused the inflammation.
A 45-year-old man comes to the emergency department due to epigastric abdominal pain and fatigue. He has had similar episodes of pain in the past. Temperature is 37.1 C (98.8 F), blood pressure is 128/69 mm Hg, pulse is 92/min, and respirations are 16/min. Physical examination shows mild epigastric tenderness and hepatomegaly. There is no jaundice or lymphadenopathy. Laboratory results are as follows: Complete blood count Hemoglobin 9.0 g/dL Mean corpuscular volume (MCV )115 µm3 Platelets 130,000/mm3 Leukocytes 5,500/mm3 CT scan of the abdomen shows pancreatic calcifications. Right upper quadrant ultrasound reveals no evidence of gallstones. Which of the following processes is most likely responsible for this patient's anemia? Diminished thymidine synthesis Impaired globin chain synthesis Impaired pyruvate decarboxylation Low rate of intracellular transamination reactions Reduced iron availability for hemoglobin synthesis
This patient's recurrent abdominal pain and pancreatic calcifications suggest chronic pancreatitis, the most common cause of which is ethanol abuse. His hepatomegaly is likely due to alcohol-associated hepatic steatosis. The patient also has macrocytic anemia (low hemoglobin and elevated mean corpuscular volume [MCV] >100 µm3), which often develops in patients with chronic alcohol use due to folic acid deficiency (within months) or vitamin B12 deficiency (within years). Both deficiencies lead to megaloblastosis, the underlying biochemical feature of which is a defect in DNA synthesis. Folic acid is a single carbon donor required for synthesis of purine and pyrimidine (eg, thymidine) bases incorporated in DNA. RNA and protein synthesis may continue relatively unaltered, leading to a state of unbalanced cytoplasmic growth with impaired cell division. Pancreatic insufficiency is also associated with vitamin B12 deficiency as pancreatic enzymes normally cleave R factor from B12, allowing B12 to bind to intrinsic factor and be absorbed. (Choice B) The thalassemias are disorders presenting with reduced or absent globin chain production. Almost all thalassemia cases result in hypochromia and microcytosis (low MCV). (Choice C) Oxidative decarboxylation of pyruvate by pyruvate dehydrogenase requires thiamine pyrophosphate (TPP), a derivative of thiamine (vitamin B1). Patients with alcohol use disorder may be prone to thiamine deficiency and could therefore suffer from impaired pyruvate decarboxylation, but this does not cause macrocytic anemia. (Choice D) Vitamin B6 (pyridoxine) catalyzes the transaminase reactions by acting as a carrier of the amino group. B6 deficiency can cause an anemia that is hypochromic, microcytic (low MCV), and sideroblastic. (Choice E) Iron deficiency anemia is typically hypochromic and microcytic (low MCV <80 µm3). Educational objective: The underlying biochemical feature of megaloblastosis is a defect in DNA synthesis. In patients with chronic alcohol use, megaloblastic macrocytic anemia can result from a nutritional deficiency of vitamin B12 or of folate, which impairs synthesis of purine and pyrimidine bases.
A 12-month-old boy is brought to the office by his father due to prolonged bleeding from mouth after slipping and hitting his face on a coffee table. He was born at term, and his mother died shortly after his birth due to bleeding complications following a home delivery. Vital signs are normal. He is awake and alert. There is blood oozing from his gums and marked bruises along his trunk and thighs. The remainder of the physical examination is normal. Laboratory evaluation shows decreased von Willebrand Factor antigen levels and activity, measured via the ristocetin cofactor assay. Which of the following sets of laboratory values is most likely present in this patient? Platelets, PT, PTT -, normal, normal -, +, + normal, normal, + normal, +, normal normal, +, +
Von Willebrand disease (vWD) is caused by genetic mutations that lead to decreased quantity or impaired function of von Willebrand factor (vWF), a multimeric glycoprotein produced by platelets and endothelial cells. VWF has 2 important hemostatic functions: It binds platelets to vascular subendothelial components at sites of injury (and platelets to each other), which results in the formation of hemostatic platelet plugs. Therefore, patients with VWD have impaired platelet aggregation but normal platelet count (as platelet consumption/production is unaffected). VWF acts as a carrier protein for factor VIII and protects it from degradation. Because factor VIII is part of the intrinsic (contact activation) coagulation pathway, patients with VWD sometimes have prolonged PTT (due to increased destruction of factor VIII), but PT will be normal. VWD is typically transmitted in an autosomal dominant fashion so a family history of bleeding is often an important diagnostic clue. Most cases are asymptomatic, but a minority (~1%) have easy bruising, skin bleeding, and bleeding from mucous membranes (eg, oropharyngeal, gastrointestinal, uterine). Testing for vWF level and activity (eg, ristocetin cofactor assay) can help confirm the diagnosis. (Choice A) Primary immune thrombocytopenia (ITP) is an autoimmune disease associated with immune-mediated platelet destruction. Although patients with ITP often have easy bleeding, VWF levels and activity are normal, and a family history of bleeding is uncommon (as ITP is an acquired disorder). ITP is marked by thrombocytopenia and normal PT/PTT. (Choice B) Disseminated intravascular coagulation is caused by widespread activation of coagulation and fibrinolysis due to sepsis, malignancy, trauma, or obstetric complication. It is an acquired disorder, so a family history of bleeding is uncommon. Disseminated intravascular coagulation is associated with low platelets and elevated PT and PTT. (Choice D) Vitamin K deficiency and warfarin use impair synthesis of vitamin K-dependent clotting factors (II, VII, IX, and X), which leads to prolonged PT and normal platelet count and PTT (PTT can sometimes be mildly elevated). Although vitamin K deficiency is common in newborns, it is very uncommon in otherwise healthy children and adults. In addition, the family history of bleeding disorder makes this much less likely than vWD. (Choice E) Patients with severe liver disease often have impaired hepatic synthetic function and are unable to generate sufficient quantities of clotting factors. Therefore, they often have elevated PT and PTT (with normal or low platelets). This otherwise healthy boy is unlikely to have significant liver disease. Educational objective: Von Willebrand disease is the most common inherited bleeding disorder and is usually transmitted in an autosomal dominant fashion. Symptomatic patients typically present with easy bruising and mucosal bleeding. Laboratory evaluation will show normal platelets, normal PT, and either normal or prolonged PTT due to low levels of factor VIII.
A 17-year-old girl comes to the office for follow-up on anemia. She has taken iron supplements regularly since being diagnosed with iron deficiency anemia 3 months ago. However, the patient still feels fatigued and does not think that the supplements have improved her symptoms. She has occasional gingival bleeding when brushing her teeth. Menses occur every 27-28 days and last 7-8 days with heavy flow, sometimes requiring her to change pads every hour. Platelet count is normal. Further evaluation reveals that the patient's platelets do not agglutinate appropriately in response to ristocetin. When normal plasma is added to the solution of patient platelets and ristocetin, appropriate platelet agglutination occurs. Which of the following is most likely deficient in this patient? Glycoprotein Ib receptors Glycoprotein IIb-IIIa receptors Hageman factor Thromboxane A2 Von Willebrand factor
Von Willebrand factor (vWF) is an important hemostatic glycoprotein (GP) synthesized by endothelial cells and megakaryocytes. Following endothelial damage, vWF binds GP Ib receptors on the platelet membrane and mediates platelet adhesion to subendothelial collagen. Deficiency of vWF impairs platelet adhesion and can lead to easy bruising and prolonged mucocutaneous bleeding (eg, gingival bleeding, heavy menses). Laboratory workup in vWF deficiency reveals a normal platelet count and an abnormal ristocetin cofactor assay. The ristocetin cofactor assay measures in-vitro, vWF-dependent platelet agglutination (indicative of impaired platelet adhesion in-vivo). Ristocetin activates GP Ib receptors on platelets and makes them available for vWF binding; when the vWF level is decreased, there is poor platelet agglutination in the presence of ristocetin. When normal plasma that contains vWF is added, appropriate platelet agglutination occurs. VWF also serves as a carrier for factor VIII to prolong its half-life, and vWF deficiency can lead to functional deficiency of factor VIII that further contributes to bleeding complications. PTT may be normal or prolonged depending on the degree of factor VIII impairment. Combined oral contraceptives are often used for treatment of menorrhagia due to vWF deficiency. Patients can also be treated with desmopressin, which stimulates vWF release from endothelium. (Choice A) Bernard-Soulier syndrome (hereditary deficiency of GP Ib receptors) is characterized by thrombocytopenia, enlarged platelets, and mucocutaneous bleeding. Platelet agglutination to ristocetin will be abnormal and, because the deficiency is with GP Ib receptors and not vWF, the addition of normal plasma will not correct the agglutination. (Choice B) Hereditary deficiency of GP IIb/IIIa receptors occurs in Glanzmann thrombasthenia, which manifests with mucocutaneous bleeding. Platelet agglutination in response to ristocetin is normal because the levels of vWF and GP Ib receptors are normal. (Choice C) Congenital deficiency of factor XII (Hageman) causes marked PTT prolongation; however, it does not cause clinical bleeding complications. Instead, patients may have a tendency for thromboembolic complications for unclear reasons. (Choice D) Thromboxane A2 deficiency is associated with aspirin treatment due to irreversible inactivation of cyclooxygenase in platelets. The major effect is on GP IIb/IIIa-mediated platelet aggregation rather than GP Ib-mediated platelet adhesion; therefore, platelet agglutination to ristocetin is normal. Educational objective: Following endothelial damage, von Willebrand factor (vWF) binds glycoprotein Ib receptors on platelets to mediate platelet adherence. The ristocetin cofactor assay measures platelet agglutination via binding of glycoprotein Ib receptors to vWF; it will be abnormal in vWF deficiency but will correct with the addition of normal (vWF-containing) plasma.
A 45-year-old-woman comes to the office for follow-up 2 weeks after hospitalization for acute pyelonephritis. The patient was treated with morphine for pain and a course of broad-spectrum antibiotics, which she has completed. Her symptoms have now resolved. The patient also has a history of mechanical mitral valve replacement and takes warfarin. Physical examination shows no new findings. Her INR was 2.5 at the time of hospital discharge but is 6.2 today despite no change to warfarin dosage. Which of the following best explains the laboratory findings in this patient? Decreased production of thromboxane A2 Disruption of intestinal bacterial flora Formation of anticlotting factor antibodies Increased cytochrome P-450 activity Increased dietary vitamin K absorption
Warfarin is a vitamin K antagonist that inhibits epoxide reductase in the liver, thereby preventing gamma carboxylation of vitamin K-dependent clotting factors (II, VII, IX, X). This creates an anticoagulant state that is reflected on laboratory assessment as a prolonged INR. Because the effect of warfarin is significantly altered by certain foods and medications, patients on long-term warfarin are at risk for subtherapeutic or supratherapeutic INR due to any of the following: Change in dietary vitamin K intake: Increase in dietary vitamin K increases the availability of vitamin K in the liver, which counters the inhibitory effect of warfarin on epoxide reductase. Therefore, patients on warfarin are advised to eat a consistent amount of vitamin K-containing foods (eg, leafy greens, brussels sprouts). Disruption to intestinal flora: Intestinal flora produce vitamin K as a byproduct of metabolism. Therefore, antibiotics that target gram-negative bacteria such as metronidazole, macrolides, and fluroquinolones can cause supratherapeutic INR due to destruction of vitamin K-producing bacteria in the gut. In this patient, the administration of an antibiotic (eg, ciprofloxacin) for pyelonephritis likely destroyed gram-negative bacteria in her gut, thereby reducing vitamin K production and causing a supratherapeutic INR. Alteration to cytochrome P450 2C9 activity: Several medications induce or inhibit cytochrome P450 2C9, which alters the speed of warfarin metabolism. For instance, phenytoin, carbamazepine, and rifampin increased cytochrome P450 2C9 activity, which can result in a subtherapeutic INR (Choice D). In contrast, metronidazole, fluconazole, and amiodarone inhibit cytochrome P450 2C9 activity, which can result in a supratherapeutic INR. (Choice A) Aspirin exerts an anticoagulation effect by inhibiting the release of thromboxane A2 by platelets, which subsequently reduces platelet aggregation. This can lead to prolonged bleeding time but does not increase INR. (Choice C) Anticlotting factor antibodies can occasionally occur due to recent pregnancy, malignancy, or rheumatologic disease. Most cases involve inhibition of factor VIII, which leads to a bleeding diathesis and prolonged aPTT (not INR, which reflects PT). This patient with a recent infection treated with antibiotics is more likely to have supratherapeutic INR due to disruption of vitamin K production by intestinal flora. (Choice E) Increased dietary vitamin K leads to a subtherapeutic INR due to increased vitamin K availability in the liver. Educational objective: Patients on warfarin who take antibiotics that target gram-negative microorganisms (eg, metronidazole, macrolides, fluroquinolones) can develop supratherapeutic INR due to disruption of vitamin K-producing intestinal flora.
A 58-year-old man comes to the emergency department due to wound dehiscence. The patient had a left subclavian catheter port placed 3 months ago to begin bevacizumab infusions for recurrent glioblastoma. The incision wound closed completely but has now partially reopened. Vital signs are normal. Examination of the left side of the chest reveals that 50% of the incision has opened; subcutaneous tissue and part of the port are visible, but there is no drainage from the wound or erythema of the surrounding skin. A complication of bevacizumab is suspected. Which of the following is the most likely primary mechanism of this patient's wound dehiscence? Depelation of endothelial growth factors for angiogenesis Eccentric myointimal proliferation and luminal obstruction of the arterioles Impaired cytoskeleton microtubule development Impaired inflammatory cell migration to the wound site Inadequate hydroxylation of collagen peptides
Wound dehiscence is partial or complete separation of previously approximated wound edges. It occurs due to disruption of the wound-healing process and may be a complication of medications such as bevacizumab. Bevacizumab is a monoclonal antibody that binds vascular endothelial growth factor (VEGF), preventing it from binding to its cell surface receptor. This inhibits angiogenesis—the process by which new blood vessels sprout from surrounding, uninjured vessels and grow into the wound—because angiogenesis is largely stimulated by VEGF. Because bevacizumab suppresses the growth of new blood vessels, it limits the blood supply to the tumor and is often referred to as cancer-starving therapy. However, because it also suppresses angiogenesis in healing wounds, it can cause wound dehiscence. The lack of angiogenesis leads to inadequate blood supply to support collagen production during the proliferation phase (3 days to 5 weeks) and collagen remodeling and cross-linking during the maturation phase (3 weeks to 2 yr). When these are inhibited, the tensile strength of the wound remains low, and dehiscence can occur even weeks to months after initial wound closure. (Choice B) Radiation therapy, commonly used in cancer treatment, causes DNA damage, resulting in apoptosis of susceptible (eg, highly proliferative) cells. However, it also damages surrounding noncancerous tissues, including local blood vessels, which can subsequently develop progressive eccentric myointimal proliferation and luminal obstruction. This can eventually result in tissue ischemia (eg, chronic skin ulcers). (Choice C) Vinca alkaloids such as vincristine and vinblastine inhibit cytoskeleton microtubule development. This results in the arrest of cell division and replication at the metaphase stage, leading to failure of division and cell death. (Choice D) Anti-inflammatory agents such as glucocorticoids can impair inflammatory cell migration to the wound site. Suppression of inflammation, the wound-healing phase that precedes proliferation, can delay the progression of wound healing. (Choice E) Vitamin C deficiency can result in inadequate hydroxylation of proline and lysine residues during collagen synthesis. Because hydroxyproline and hydroxylysine are essential for collagen cross-linking, vitamin C deficiency compromises collagen cross-linking and reduces the maximal tensile strength of the wound. Educational objective: Bevacizumab is a monoclonal antibody that inhibits the binding of vascular endothelial growth factor (VEGF) to its receptor. This inhibits angiogenesis and limits blood supply to tumors (ie, cancer-starving therapy). However, it also impairs wound healing and can lead to wound dehiscence.
Further evaluation shows decreased von Willebrand factor activity in the patient's serum. This protein normally binds to which of the following? Collagen Fibrin polymer Prostacyclin Protein C Thrombin
von Willebrand factor (vWF) serves 2 important roles in hemostasis: vWF promotes platelet adhesion at sites of vascular injury by binding to and crosslinking platelet glycoproteins (primarily GpIb) with exposed collagen underneath damaged endothelium. vWF also enhances platelet aggregation, particularly under conditions of high shear stress (such as in small vessels). vWF functions as a protective carrier protein for factor VIII that increases its plasma half-life. In the absence of vWF, factor VIII is rapidly degraded in the circulation via proteolytic inactivation. (Choice B) A hemostatic fibrin polymer is the end result of the coagulation cascade. Fibrin also plays a role in inflammation and coagulation regulation. (Choice C) Prostacyclin functions both as an inhibitor of platelet aggregation and a vasodilator. (Choice D) Protein C is a vitamin K-dependent anticoagulant activated by bound thrombin in the presence of endothelial thrombomodulin. Protein C acts together with protein S to inactivate the activated forms of factors V and VIII, which are necessary for the production of thrombin. (Choice E) Thrombin is produced from prothrombin by activated factor X. Thrombin, in turn, converts fibrinogen to fibrin. Thrombin also activates factors V, VIII, and XIII. Educational objective: von Willebrand factor (vWF) functions as a promoter of platelet adhesion at sites of vascular injury by binding platelet glycoproteins to subendothelial collagen on injured blood vessel walls. It also acts as a protective carrier protein for circulating factor VIII.